Вы находитесь на странице: 1из 60

VERSION 6.

EXAM CONTENT
MANUAL
APICS Certified in Production and
Inventory Management
Exam Content Manual

Version 6.0
Visit apics.org/ecmerrata for APICS CPIM Exam Content Manual errata.
Internet links cited in the bibliographic references can be found in a more usable format on the
APICS website at apics.org/cpim.

The references in this manual have been selected solely based on their educational value to the
APICS CPIM certification program and the content of the material. APICS does not endorse any
service or other materials that may be offered or recommended by the authors or publishers of
books and publications listed in this manual.

©2017 APICS

8430 West Bryn Mawr Avenue, Suite 1000


Chicago, IL 60631-3439 USA
Phone: 1-800-444-2742 or +1-773-867-1777
Fax: +1-773-639-3000

No portion of this document may be reproduced under any circumstances. CPIM is a registered
trademark of APICS.

Stock #09051-V60

09051-V60
Table of Contents
Letter to Candidates ................................................................................................................ 1
Introduction .............................................................................................................................. 3
About the APICS CPIM Examinations ...................................................................................... 3
Question Format ............................................................................................................................... 3
Taking the Test .................................................................................................................................. 4
Interpreting Test Scores ................................................................................................................... 4
APICS Certified Fellow in Production and Inventory Management (CFPIM) ......................... 4
Studying for the APICS CPIM Exam ......................................................................................... 4
APICS CPIM References .................................................................................................................... 5
Terminology ....................................................................................................................................... 5
Additional Resources for APICS CPIM Candidates ................................................................ 5
APICS CPIM Learning System ........................................................................................................... 5
APICS CPIM Instructor-Led Review Courses .................................................................................... 6
APICS Educational Programs............................................................................................................ 6
Exam Content vs. Courseware ................................................................................................ 6
APICS CPIM Certification Maintenance: Continuing Professional Development ................. 7
The Importance of Certification Maintenance ................................................................................ 7
APICS Code of Ethics ............................................................................................................... 8
Bibliography and References for CPIM Exams ....................................................................... 9
Key Terminology ..................................................................................................................... 11
APICS CPIM Part 1.................................................................................................................. 23
CPIM Part 1 Sample Questions ............................................................................................. 29
APICS CPIM Part 2.................................................................................................................. 32
CPIM Part 2 Sample Questions ............................................................................................. 47
Answers to Sample Questions .............................................................................................. 50

CPIM Exam Content Manual i


The APICS CPIM program utilizes two exams in
Letter to Candidates a progression from basic - CPIM Part 1, to
advanced - CPIM Part 2. It is strongly
Dear Candidate:
recommended that candidates complete CPIM
Part 1 before tackling the more advanced
For more than four decades, the Certified in
topics in CPIM Part 2. If you follow this
Production and Inventory Management (CPIM)
sequence of the two exams, you will increase
program has been recognized as the
your understanding and probability of success.
international standard for individual
assessment in the field of operations
The following is a summary of each of the two
managements as it relates to transformation
APICS CPIM exams:
of products and services. Initiated by APICS in
1973, it continues to provide a standard for
APICS CPIM Part 1 (previously known as
individuals and organizations to evaluate their
Basics of Supply Chain Management)
knowledge of this ever-evolving field. APICS
As the entry-level learning program, CPIM Part
has administered more than 1 million exams
in over 40 countries, and more than 105,000 1 introduces the material presented in depth
professionals have earned the APICS CPIM in CPIM Part 2.
designation with 3,000 members at the Fellow
level. Part 1 introduces the definitions and concepts
for planning and controlling the flow of
The mission of the APICS CPIM program is to products and services into, through and out of
be the premier professional certification for an organization. Many of the key terms in Part
supply chain and operations management. 1 are expanded upon in Part 2. Part 1 explains
The exams test the candidate’s knowledge fundamental relationships among the various
and understanding of the principles and activities that may occur in the supply chain
practices of operations and inventory network from suppliers to customers. In
management. The APICS CPIM program is addition, Part 1 covers types of manufacturing
designed to educate individuals in the various and service systems, forecasting, master
concepts, methodologies, terminology, and planning, material requirements planning,
integration of topics within the supply chain capacity management, production activity
and operations management functions. Each control, purchasing, inventory management,
exam tests candidates’ in-depth knowledge of and distribution. Four key management
these concepts. APICS has worked to ensure philosophies are covered: enterprise
that CPIM exams are consistently reliable and resources planning, lean, quality
the highest professional standards are used management, and the theory of constraints.
to develop and administer the program.
APICS CPIM Part 2
Because organizations operate in a changing CPIM Part 2, combines key concepts from the
and challenging international environment, field of operations management. It details four
the APICS body of knowledge continues to areas of knowledge:
grow and include recognized concepts and
tools for improved organizational Master Planning of Resources (MPR)
competitiveness and effectiveness. The CPIM Detailed Scheduling and Planning (DSP)
Exam Content Manual (ECM) is regularly
updated to reflect changes in the body of Execution and Control of Operations (ECO)
knowledge and assist candidates in their Strategic Management of Resources (SMR)
understanding of the scope of material
covered.

CPIM Exam Content Manual 1


In Master Planning of Resources, candidates The APICS CPIM program continues to evolve,
explore processes used to develop sales and incorporating relevant and current concepts
operations plans, identify and assess internal and techniques into the body of knowledge.
and external demand management Examples include, supply chain management,
requirements, forecasting, planning and lean, service industries, globalization, theory
replenishment in a distribution environment, of constraints, integrated business planning,
and gain an understanding of developing a sales and operations planning, outsourcing,
feasible master schedule. The section focuses critical chain, and sustainability. APICS CPIM
on a plan of supply; relating management of is an outstanding educational program. APICS
demand to manufacturing, distribution and relies on your comments and suggestions to
service environments; and developing and maintain and improve the program for future
validating the master schedule. candidates. We wish you success in your
pursuit of CPIM certification.
In Detailed Scheduling and Planning,
candidates focus on the various techniques
for inventory, procurement, and material and
capacity scheduling. This section includes
detailed descriptions of material requirements
William R. Leedale, CFPIM, CIRM, CSCP, CLTD
planning, capacity requirements planning,
Chair, CPIM Subcommittee
inventory management practices, and
procurement and supplier planning.
Candidates will also become familiar with
sustainable practices, supplier partnerships,
lean principles, and outsourcing strategies
and techniques.

In Execution and Control of Operations


candidates focus on four main areas:
execution of operational plans and schedules,
control of the work completed and the
analysis of results, the management of and
communication in the workplace, and the
importance of utilizing appropriate design
principles. The section explains techniques for
scheduling and controlling operations within
available capacity. It also deals with the
execution of quality initiatives, cost
management, problem solving, and
continuous improvement.

In Strategic Management of Resources,


candidates explore the relationship of existing
and emerging processes and technologies to
operations strategy and supply chain-related
functions for manufacturing and service
organizations. The section addresses three
main topics: understanding the business
environment, developing operations strategy,
and implementing operations strategy.

2 ©APICS
Sufficient references are given for each
Introduction topical area to provide different approaches to
material covered in each exam and different
This Exam Content Manual (ECM) provides
styles of presenting it. Reading the available
assistance for those preparing for the CPIM
APICS Magazine will help you maintain an
certification examinations. The objective of
awareness of changes in the state of this
this manual is to outline the APICS CPIM body
discipline. Attending APICS conferences as
of knowledge, which the APICS Certification
well as local Chapter professional
Committee has organized into two exams:
development meetings is also a way to keep
current on the CPIM body-of-knowledge.
 CPIM Part 1 (previously known as Please visit our website, apics.org, for details
Basics of Supply Chain Management) regarding APICS conferences and locating
 CPIM Part 2 your local chapter.

In this manual, each exam description begins


with a statement on the scope of the subject
About the APICS CPIM
matter, followed by a descriptive outline of the Examinations
content. References selected by the CPIM
exam committees can be found on pages 9- Each of the two APICS CPIM exams consist of
10. Key terminology for each exam is provided 150 questions, of which 20 are pre-test
on pages 11-22. Each exam overview questions that do not contribute to the total
concludes with sample questions typical of score, but are necessary for research
those that appear on the examinations. The purposes. There is a three and one-half hour
correct answers for the sample questions, time limit for each APICS CPIM exam. For
with brief explanations of why they are correct, more information regarding testing and
appear at the end the manual. registration policies and procedures, please
download the APICS Exam Handbook from the
The recommended procedure for mastering APICS website at apics.org/handbook, or call
the subject matter is to review the content APICS Customer Service at 1-800-444-2742
outline, which defines the material, and then (United States and Canada) or +1-773-867-
to study each topic, using the references 1777.
identified. At the end of each major section is
a list of the references that apply to the topics
in that section. The first number indicates the
Question Format
sequence number for the reference in the The questions on the CPIM examinations are
Bibliography section on pages 9-10, and the intended to test a candidate’s understanding
numbers in parentheses indicate the of the CPIM body of knowledge and frequently
chapter(s) within that selected reference. require the candidate to select the best of four
These outlines form the content and structure choices, complete a calculation based on the
for the certification examinations. Candidates information given, or they may ask the
should understand the definitions of the key candidate to illustrate their understanding of a
terms in the content outline, as well as the concept, process, or procedure. These
outlined techniques. Additionally, candidates questions may require the examinee to make
should know why and how to apply techniques finer distinctions than the exercises or items
and which ones to select for different presented in a course. It is helpful to
situations. understand the various formats of questions
on the examination. For sample questions and
an introduction to Technology Enhanced Items
found in the CPIM Part 1 exam, please see
pages 29–31. CPIM Part 2 sample questions
can be found on pages 47–49.

CPIM Exam Content Manual 3


The CPIM exams scaled score range is 200 –
Taking the Test 350.
The key to success is a thorough
understanding of the subject matter as 200–299: Fail
outlined in this ECM. All questions are based
on the current CPIM body of knowledge as 300–350: Pass
defined in the exam content manual.
320 and greater: Fellow level
When you begin each exam, read all the For each examination, candidates receive a
directions carefully. Be sure you understand score for the total exam. All candidates will
the directions before you begin to answer any also receive diagnostic information by topic
questions. areas on their performance.

Read each question carefully. If a question


includes additional material, such as a table, APICS Certified Fellow in
graph, or situation, be sure to study it before Production and Inventory
answering the question and do not assume you
know what is being asked without reading the Management (CFPIM)
question completely. Every effort has been
made to avoid misleading wording and to The distinguishing characteristic of a Certified
provide sufficient information for each question. Fellow in Production and Inventory Management
Choose the best answer from the choices (CFPIM) is the willingness to share acquired
given. Do not look for hidden tricks or knowledge with others through presenting,
exceptions to the norm. For each question, teaching, publishing, and participating in APICS
one and only one of the answer choices educational activities. This knowledge sharing
represents the correct answer. must take place above and beyond a candidate's
normal job duties and be directly related to the
Once you begin the test, approach the questions APICS CPIM body of knowledge.
in order, but do not waste time on those that are
unfamiliar or seem difficult to you. Go on to the To obtain the APICS CFPIM designation, an
other questions and return to the difficult ones application form must be completed and
later if time allows. If you have some knowledge submitted to the APICS corporate office. Points
about a particular question, you may be able to are awarded based on the following criteria:
eliminate one or more choices as incorrect. Your APICS CPIM exams passed, presentations, high
score on the test will be based on the number of scores on CPIM exams (320 or greater),
questions you answer correctly, with no penalty published works, classroom teaching, and
for incorrect answers; therefore, it is to your various volunteer or practitioner activities.
advantage to guess rather than not answer a To apply for the CFPIM Certification, please visit
question. Avoid changing an answer unless you apics.org/cfpim.
are absolutely certain that you marked the
wrong answer. Studying for the APICS CPIM
Exams
Interpreting Test Scores
Scoring is based on your correct responses. APICS offers several resources to help
There is no penalty for incorrect answers. The individuals prepare for the APICS CPIM
omission of an answer will be counted the examinations.
same as an incorrect answer.

4 ©APICS
and the certification exam committees have
APICS CPIM References worked to provide consistency with preferred
Bibliography. The APICS CPIM Examination
terminology. However, synonyms are often
Subcommittees have identified several
used by authors in the various references
references for the APICS CPIM exams. These
used to compile the body of knowledge.
are listed in the Bibliography section of this
Candidates are encouraged to be familiar with
manual on pages 9-10. All the references
all terms and concepts listed within the
contain excellent material that will assist in
outline and key terminology section, using the
test preparation. For additional information on
APICS Dictionary as the primary guide for
the APICS CPIM references, visit the APICS
definitions.
website at apics.org/CPIM, or call APICS
Customer Service at 1-800-444-2742 (United
States and Canada) or +1-773-867-1777. Additional Resources for
A candidate may discover that the material
covered in the chapters of one reference
APICS CPIM Candidates
duplicates material covered in another
In addition to the cited references, it may be
reference. Both sources are included as
helpful for you to pursue chapter-sponsored
references to allow candidates discretion in
selecting test preparation materials that they courses, college courses, APICS workshops,
find most accessible and understandable. self-study courses, or courses offered by the
APICS network of authorized education
In deciding if a single reference is sufficient, providers (AEPs) as a means of learning the
body of knowledge tested in the certification
candidates should assess their own level of
program. A wide variety of courses and
knowledge against both the descriptive
materials are available. As with any
examination specifications and the detailed
topic list in the content outline. If there are investment, you should research various
any areas of weakness, the candidate should options before making your preparation
consult other references as part of the test decisions.
preparation process.
APICS CPIM Learning
CPIM Content outlines. The content outlines
for CPIM Parts 1 and 2 provided in this ECM
Systems
should be considered a primary resource for The APICS CPIM Learning Systems for Parts 1
exam preparation. It provides an overview of and 2, are comprehensive professional
the major topics included in the exam, as well development and certification preparation
as a list of the concepts relevant to that topic. programs. The Learning Systems include a
copy of the Exam Content Manual and utilize
APICS Dictionary. The APICS Dictionary is an all the APICS CPIM references, including those
essential reference to the exam content listed in the Bibliography section and the
manual and APICS exams. Within the APICS Dictionary. Additional outside
profession, terminology varies among references are also used to supplement ECM
industries, companies, and the academic topics. These systems combine reading
community. The examination uses standard material with interactive online tools, and are
terminology as defined in the APICS available in self-directed and instructor led
Dictionary. Recognizing the terms and formats.
understanding their definitions are essential.
The APICS CPIM Learning Systems do not
“teach the tests” and in many areas, review
Terminology the concepts rather than teach the concepts.
In studying for the APICS CPIM certification There will likely be some content in the APICS
exam, candidates may discover multiple terms CPIM Learning Systems not covered by the
used to denote the same technique. APICS exams; conversely, there will likely be some

CPIM Exam Content Manual 5


content in the exam not covered by the four choices, complete a calculation based on
Learning System. Thus, it is essential for the information given, or they may ask the
candidates to use the current exam content candidate to illustrate their understanding of a
manual in their studies. concept, process, or procedure. These
questions may require the candidate to make
finer distinctions than the exercises or items
APICS CPIM Instructor-Led presented in a review course.
Review Courses
The instructor-led formats for CPIM Parts 1 APICS uses a rigorous process for creating its
and 2 combine the APICS CPIM Learning Certification exams and courseware. Exam
Systems reading material and online and courseware study materials are
components with the leadership of a qualified developed separately to maintain the integrity
instructor; peer collaboration; company of the exam process.
networking; and a structured schedule to keep
participants on track. Learn more about APICS APICS exam committees define the contents
recognized instructors at of the Exam Content Manual (ECM), which
apics.org/recognizedinstructors or find local determines the areas that will be tested in
APICS partners that provide APICS CPIM APICS certification exams. The ECM defines
courses at apics.org/finder or call APICS the body of knowledge that can be tested and
Customer Service at 1-800-444-2742 (United every exam question is linked to the ECM
States and Canada) or +1-773-867-1777 to content. The APICS exam committees also
obtain contact information for your local select the references that will be used for
chapter. For courses outside North America, exam development and they create and
visit apics.org/international to locate the maintain exams.
nearest partner.
A separate courseware committee and a third-
party vendor create the courseware using the
APICS Educational Programs ECM and the recommended references.
In addition to the annual APICS International
Conference & Expo, APICS offers a variety of
Courseware developers and/or instructors
educational programs. For details regarding
may believe that additional material needs to
APICS learning opportunities and information
be taught or included to ensure understanding
on course availability, visit apics.org or call
of the body of knowledge. They also may
APICS Customer Service at 1-800-444-2742
decide that a concept or term is adequately
(United States and Canada) or +1-773-867-
covered by the definitions in the APICS
1777. For a complete list of APICS resources,
Dictionary or content outlines and not cover it
visit the APICS website at apics.org or call
in the course. These differences sometime
APICS Customer Service at 1-800-444-2742
lead candidates to perceive a disconnect
or +1-773-867-1777.
between the courseware and the exam, when
in fact they are both covering the same body
Exam Content vs. Courseware of knowledge.

Certification has a very different purpose than Question and answer sets for APICS exams
education. It is to determine whether a are written by exam committee members, who
candidate meets a minimum set of are subject matter experts, and volunteers
requirements in relation to a body of who have earned APICS Certification
knowledge. Certification examinations test an designations. The exam committees must
individual’s knowledge and ability to apply identify the specific entry in the ECM that is
that knowledge to specific situations. being tested and one or more of the
Certification exam questions frequently references listed in the ECM that supports the
require the candidate to select the best of the correct answer. All exam questions and

6 ©APICS
answers are reviewed and typically revised by
APICS exam committee members. Exam
APICS CPIM Certification
committees, APICS Test Development staff, Maintenance: Continuing
and a third-party exam development
contractor, all review the potential test
Professional Development
questions for correctness of form, spelling,
and grammar. The Importance of
A potential test question may be reviewed
Certification Maintenance
The growing number of individuals choosing to
multiple times before it actually appears on an pursue professional development through the
examination. Potential test questions initially APICS CPIM program indicates a strong
appear on exams in what is referred to as awareness that continuing education and
“pretest” status, in order to collect statistics skills development are essential to meeting
on the questions. the information and technological challenges
in today’s rapidly evolving workplace and
Because each test form has a limited number global marketplace. Professional development
of questions, it samples representative areas opens doors to individual career opportunities
of the body of knowledge as defined by the and organizational success.
ECM. While each test form is different, all
areas tested are contained within the body of Although APICS CPIM recognition and
knowledge as defined by the ECM. maintenance are voluntary programs, they
equally demonstrate one’s commitment to
The following graphic is representation of the achieving the highest level of professional
type of relationship between the Operations development and standards of excellence.
Management Body of Knowledge (OMBOK),
courseware / Learning Systems, ECM’s and The APICS CPIM certification maintenance
different examinations. program upholds both the objectives of the
APICS CPIM program and the APICS vision to
promote lifelong learning. This flexible
program recognizes that individuals are at
various levels in their careers, come from
many industries, have different educational
needs and career goals, and have varying
access to continuing education. Thus,
requirements for maintaining certification can
be met through multiple sources and a variety
of professional development activities. These
sources and activities are intended to help
prepare for the challenges ahead and
maintain a professional edge by:

 preserving the currency of hard-


earned certification credentials
 expanding your knowledge of the
latest industry practices
 exploring new technology solutions
 reinforcing skills
 improving job performance
 demonstrating commitment to
excellence
 increasing competitive advantage

CPIM Exam Content Manual 7


To ensure ongoing professional growth and  In your professional capacity, respect
lifelong learning, APICS CPIM designees must the fundamental rights and dignity of
complete the certification maintenance all individuals. You must demonstrate
program every five years. For complete details sensitivity to cultural, individual, and
on how to maintain your APICS CPIM role differences, including those due
designation, please visit to age, gender, race, ethnicity,
apics.org/maintenance. national origin, religion, sexual
orientation, disability, language, and
APICS Code of Ethics socio-economic status;
 In your professional capacity, not
engage in behavior that is harassing
When you start an examination, you will be or demeaning to others based on
asked to pledge to abide by the APICS Code of factors including, but not limited to,
Ethics. Once certified, you pledge to continue age, gender, race, ethnicity, national
your education to increase your contribution to origin, religion, sexual orientation,
the supply chain management profession. disability, language, or socio-economic
After achieving the APICS CPIM designation, status;
you pledge also to share your knowledge with
 Adhere to this Code of Conduct and its
others by participating in APICS research and
application to your professional work.
educational activities at local, district,
Lack of awareness or
national, and international levels.
misunderstanding of an ethical
standard is not itself a defense to a
APICS Code of Ethics is as follows:
charge of unethical conduct;
 Maintain exemplary standards of
 Contact the Ethics Committee when
professional conduct;
uncertain whether a particular
 Not misrepresent your qualifications, situation or course of action violates
experience, or education to APICS or the Code of Conduct; and
others you serve in a professional
 Not to become the subject of public
capacity;
disrepute, contempt, or scandal that
 Respect and not violate the United affects your image or goodwill.
States Copyright of all APICS
materials, including but not limited to Failure to abide by APICS Code of Ethics policy
courseware, magazine articles and may result in sanctions up to and including
other APICS publications, APICS decertification.
conference presentations, and CPIM,
CSCP, CLTD, and SCOR-P examination
resources. In this same spirit, you
must not violate the copyright of other
organizations and individuals in your
professional capacity;
 Not engage in or sanction any
exploitation of one’s membership,
company, or profession;
 Encourage and cooperate in the
interchange of knowledge and
techniques for the mutual benefit of
the profession;

8 ©APICS
Bibliography and References for CPIM Part 1 and Part 2
These references cover the material a successful CPIM candidate is required to know.

References Author(s) Part 1 Part 2


1 APICS Dictionary, 15th ed., 2016 APICS
X X

2 APICS CPIM Detailed Scheduling and Planning APICS Exam


X
Reprints, 2010 Committee
3 APICS CPIM Execution and Control of APICS Exam
X
Operations Reprints, 2015 Committee
4 APICS CPIM Master Planning of Resources APICS Exam
X
Reprints, 2010 Committee
5 Accounting Handbook, 6th ed. 2015 Shim, J. K.,
Siegal, J. G.,
X
Dauber, N.,
Qureshi, A. A.
6 Crafting and Executing Strategy: The Quest for Thompson, A. A.,
Competitive Advantage: Concepts and Peteraf, M. A.,
X
Readings, 20th ed., 2016 Gamble, J. E.,
Strickland III, A. J.
7 Designing and Managing the Supply Chain: Simchi-Levi, D.,
Concepts, Strategies and Case Studies, Kaminsky, P., X
3rd ed., 2008 Simchi-Levi, E.
8 Distribution Planning and Control: Managing in Ross, D. F.
the Era of Supply Chain Management, X
3rd ed. 2015
9 The G4 Sustainability Reporting Guidelines: Global Reporting
Reporting Principles and Standard Disclosures, Initiative X
2015
10 Introduction to Materials Management, Chapman, S. N.,
8th ed., 2017 Arnold, J.R. Tony,
X X
Gatewood, A. K.
Clive, L. M.
11 Juran’s Quality Handbook: The Complete Guide DeFeo, J. A.
to Performance Excellence, X
7th ed., 2017
12 Leading Change, 2012 Kotter, J. P. X
13 Lean Production Simplified: A Plain-Language Dennis, P.
Guide to the World’s Most Powerful Production X X
System, 3rd ed., 2015
14 The Lean Toolbox: A handbook for lean Bicheno, J.,
transformation, 5th ed., 2016 Holweg, M. X

CPIM Exam Content Manual 9


References Author(s) Part 1 Part 2
15 Making Sustainability Work: Best Practices in Epstein, M. J.,
Managing and Measuring Corporate Social, Buhovac, A. R.
Environmental and Economic Impacts, X
2nd ed. 2014
16 Manufacturing Planning and Control for Supply Jacobs, F. R.,
Chain Management, APICS/CPIM Certification Berry, W. L.,
X
Edition, 2011 Whybark, D. C.,
Vollmann, T. E.
17 Operations Strategy, 4th ed., 2015 Slack, N.,
X
Lewis, M.
18 Project Management: A System Approach to Kerzner, H. R.
Planning, Scheduling, and Controlling, X
11th ed., 2013
19 United Nations Global Compact: Corporate UN Global
X
Sustainability in the World Economy, 2014 Compact Office
20 United Nations Global Compact Management Deloitte Touche
X
Model: Framework for Implementation, 2010 Tohmatsu

Note: At the end of each major section in the CPIM Part 1 and 2 Content Outlines is a list of the
references that apply to the topics within that section. The first number indicates the sequence
number for the references designated in each subject area within the content outline. For example, 7
(chapters 4–7, 10) means the reference, Designing and Managing Supply Chain 3rd edition, 2008.
Chapters 4, 5, 6, 7 and 10 contain content relevant to that subject matter.

Content outline. The content outline for each exam provides an overview of the major topics included
in that module. Each major topic is denoted by a Roman numeral and is followed by a list of the
references that are particularly relevant to that topic.

APICS Dictionary. The APICS Dictionary, 15th edition, is an essential publication that applies to the
exam content manual and exams. Within the profession, terminology varies among industries,
companies, and the academic community. Each examination uses standard terminology as defined
in the APICS Dictionary. Recognizing the terms and understanding their definitions are essential.

Reprints. The committee responsible for the exam content manual and examination selects articles
that are particularly applicable to the curricula and exam preparation. These articles then are
reprinted in exam-specific collections. The reprints are included in the references for the CPIM Part 2
exam.

The APICS Dictionary is available from the APICS Book store; please visit apics.org/shopapics. Other
references can be found at the APICS Amazon store at, astore.amazon.com/apics01-20.

10 ©APICS
Key Terminology
Candidates are encouraged to be familiar with all key terms listed below for the corresponding exams.
The APICS Dictionary is the primary guideline for all definitions of the key terms.

In studying for the APICS CPIM certification, candidates may discover multiple terms used to denote
the same technique. Examples of this include “sales and operations planning” versus “production
planning” and “master production schedule” versus “master schedule”. APICS has attempted to
provide consistency across all both exams with recognized and preferred terminology. However,
synonyms are often used by authors in the various references used to compile the body of knowledge.

CPIM Key Terminology


Part 1 Part 2 Part 1 Part 2
14 Points (Deming’s) X assemble-to-order X
A3 method X assembly line X
ABC classification X assignable cause X X
abnormal demand X attribute data X
absorption costing X availability X
acceptable quality level available capacity X
X
(AQL) available inventory X
acceptance sampling X available time X
action message X available-to-promise (ATP) X
activation X average cost per unit X
activity-based cost average inventory X
X
accounting average outgoing quality
activity based management X
X limit (AOQL)
(ABM) back scheduling X
actual costs X backflush X
actual demand X backflush costing X
adaptive smoothing X backhauling X
adjustable capacity X backlog X
advanced planning and backorder X
X X
scheduling (APS) backward integration X
advanced planning system backward scheduling X
X
(APS) balanced scorecard X
advance ship notice (ASN) X balance sheet X
aggregate forecast X balancing operations X
aggregate plan X bar code X
agility X baseline measures X
allocation X base series X
alpha factor X basic seven tools of quality
alternate operation X X
(B7)
alternate routing X batch X X
analysis of variance (ANOVA) X batch picking X
andon X X batch processing X
anticipated delay report X benchmarking X
anticipation inventories X benchmark measures X
appraisal costs X

CPIM Exam Content Manual 11


CPIM Key Terminology
Part 1 Part 2 Part 1 Part 2
bias X cellular manufacturing X X
bill of distribution X centralized inventory control X
bill of labor X central point scheduling X
bill of lading (uniform) X certificate of compliance X
bill of material (BOM) X certification audits X
bill of resources X certified supplier X X
block scheduling X changeover X
bonded warehouse X changeover costs X
bottleneck X X chase production method X X
bottleneck operation X chase strategy X
bottom-up replanning X check sheet X
break-bulk X closed-loop MRP X
break-even point X X collaborative planning,
bucketed system X forecasting, and X
bucketless system X replenishment (CPFR)
budgeted capacity X common carrier X
buffer X X common causes X
buffer management X X common parts bill of
X
buffer stock X material
bullwhip effect X X competitive advantage X
business plan X competitive analysis X
business process component X
X concurrent design X
reengineering (BPR)
business-to-business concurrent engineering X
X
commerce (B2B) conformance X
by-product X consignment X
calculated capacity X constraint X X
capable-to-promise (CTP) X X constraints management X
capacity available X X consuming the forecast X
capacity-constrained continuous improvement X
X
resource (CCR) continuous manufacturing X
capacity control X continuous process control X
capacity management X X continuous process
X
capacity planning X improvement (CPI)
capacity planning using continuous production X X
X
overall factors (CPOF) continuous replenishment X
capacity-related costs X contract carrier X
capacity requirements X contribution X
capacity requirements contribution margin X
X
planning (CRP) control chart X X
capacity strategy X control limit X X
capacity utilization X control points X
carrying cost X co-product X
cash conversion cycle X core competencies X
cash flow X core process X
cash-to-cash cycle time X corporate culture X
cause-and-effect diagram X X corrective action X
cell X correlation X

12 ©APICS
CPIM Key Terminology
Part 1 Part 2 Part 1 Part 2
cost center X Delphi method X
cost of goods sold X demand filter X
cost of poor quality X demand forecasting X
cost of quality X demand lead time X
cost variance X demand management X
cost-volume-profit analysis X demand planning X
count point X demand time fence (DTF) X
critical chain method X demonstrated capacity X X
critical characteristics X demurrage X
critical path method (CPM) X X dependent demand X
critical point backflush X design for manufacturability X
critical ratio X design for manufacture and
X
critical-to-quality assembly (DFMA)
X
characteristics (CTQs) design of experiments (DOE) X
cross-docking X design-to-order X
cumulative available-to- detention X
X
promise deviation X
cumulative lead time X direct costs X
current ratio X direct labor X
curve fitting X direct material X
customer relationship discounted cash flow X
X X
management (CRM) discrete available-to-
customer service X X
promise
customer service level X discrete manufacturing X
customer-supplier discrete order picking X
X
partnership disintermediation X
customs broker X dispatching X
cycle counting X distressed goods X
cycle stock X distribution X X
cycle time X distribution center X X
data governance X distribution channel X X
days of supply X distribution inventory X
decentralized inventory distribution network
X X
control structure
decision matrix X distribution of forecast
decision support system X
X errors
(DSS) distribution requirements
decomposition X X
planning (DRP)
decoupling X distribution warehouse X
decoupling inventory X divergent point X
dedicated capacity X dock-to-stock X
dedicated line X downtime X
de-expedite X drop ship X
define, measure, analyze, drum-buffer-rope (DBR) X X
improve, control (DMAIC) X drum schedule X
process duty X
delivery lead time X X early manufacturing
delivery schedule X X
involvement

CPIM Exam Content Manual 13


CPIM Key Terminology
Part 1 Part 2 Part 1 Part 2
early supplier involvement first-order smoothing X
X
(ESI) first pass yield X
earned hours X fishbone analysis X
echelon X fitness for use X
e-commerce X five focusing steps X
econometric model X five forces model of
X
economic order quantity competition
X
(EOQ) five Ss X X
economic value added X five whys X
effective date X fixed cost X
efficiency X X fixed-location storage X
electronic data interchange fixed order quantity X
X
(EDI) fixed overhead X
employee empowerment X X fixed-position manufacturing X
employee involvement (EI) X X flexibility X
engineer-to-order X flexible workforce X
enterprise resources floor stocks X
X X
planning (ERP) flowchart X X
environmentally responsible flow control X
X
business flow processing X
excess capacity X flow rate X
expedite X flow shop X
explode X fluctuation inventory X
exponential smoothing focused factory X
X
forecast focus forecasting X
external failure costs X forecast X
external setup time X X forecast consumption X
extrapolation X forecast error X
extrinsic forecasting method X forecast horizon X
fabricator X forecast interval X
failsafe work methods X forecast management X
failure mode effects
X form-fit-function X
analysis (FMEA)
forward flow scheduling X
feature X
forward integration X
feedback X
forward scheduling X X
feeder workstations X
four Ps X
field service X
freight consolidation X
fill rate X
freight forwarder X
final assembly schedule
X X frequency distribution X
(FAS)
functional layout X X
finished goods inventory X
functional product X
finishing lead time X
funnel experiment X
finite forward scheduling X X
Gantt chart X X
finite loading X X
gatekeeping X
finite scheduling X
gateway work center X
firm planned order (FPO) X
gemba X
first-article inspection X
gemba walk X
first in, first out (FIFO) X

14 ©APICS
CPIM Key Terminology
Part 1 Part 2 Part 1 Part 2
genchi genbutsu X internal customer X
general and administrative internal failure costs X
X
expenses (G&A) internal rate of return X
generally accepted internal setup time X X
X X
accounting principles (GAAP) interoperation time X
global measurements X interplant demand X X
Global Reporting Initiative in-transit inventory X
X
(GRI) intrinsic forecast method X
global trade identification inventory accounting X
X
number (GTIN) inventory accuracy X
go/no-go X inventory adjustment X
green manufacturing X inventory buffer X
green reverse logistics X inventory control X
gross margin X inventory investment X
gross requirement X inventory management X
group technology (GT) X inventory ordering system X
hansei X inventory policy X
hazmat X inventory turnover X X
hedge X inventory valuation X
hedge inventory X Ishikawa diagram X
heijunka X X ISO 14000 Series
histogram X X X
Standards
horizontal dependency X ISO 9000 X
horizontally integrated firm X ISO 26000 X
hoshin X X item master record X
hoshin planning X jidoka X X
house of quality (HOQ) X jishuken X
hurdle rate X job analysis X
hybrid production method X job costing X X
hypothesis testing X job enlargement X
idle capacity X X job enrichment X
idle time X job sequencing rules X
inactive inventory X job shop X X
inbound stockpoint X job shop scheduling X X
income statement X job status X
incoterms X joint replenishment X
indented bill of material X Juran trilogy X
independent demand X Just-in-Time (JIT) X
indirect costs X kaizen X X
infinite loading X X kaizen blitz X
information system kaizen event X
X
architecture kanban X X
input/output control (I/O) X X keiretsu X
insourcing X key performance indicator
intangible costs X X X
(KPI)
intellectual property X key success factors X
intermittent production X kit X
intermodal transport X knowledge-based system X

CPIM Exam Content Manual 15


CPIM Key Terminology
Part 1 Part 2 Part 1 Part 2
labor efficiency X lower control limit (LCL) X
labor productivity X lower specification limit
X
labor standard X (LSL)
lag capacity strategy X low-level code X
landed cost X X machine center X
last in, first out (LIFO) X machine hours X
lead capacity strategy X machine-limited capacity X
leading indicator X machine loading X
lead time X X maintenance, repair, and
X X
lead-time offset X operating (MRO) supplies
lean enterprise X make-or buy decision X
lean metric X make-to-order X
lean production X X make-to-stock X
lean six sigma X management by walking
X
learning curve X around (MBWA)
learning organization X managerial accounting X
least changeover cost X manufacturing calendar X
least-squares method X manufacturing environment X
least total cost X manufacturing execution
X
level loading X systems (MES)
level of service X manufacturing layout
X
level production method X strategies
level schedule X X manufacturing lead time X X
liabilities X manufacturing order X
life cycle assessment (LCA) X manufacturing order
X
reporting
lifecycle analysis X
manufacturing philosophy X
lifecycle costing X
manufacturing process X
limiting operation X
manufacturing resource
line X X
planning (MRP II)
line balancing X
manufacturing strategy X
line haul costs X
market driven X
Little's Law X
marketing strategy X
load X
mass customization X X
load leveling X X
master planning X
load profile X
master planning of
load projection X X
resources
local measures X master production schedule
logistics X X X
(MPS)
lot X master schedule X
lot control X X master schedule item X
lot cost X master scheduler X
lot-for-lot X material-dominated
lot size X X
scheduling (MDS)
lot-size inventory X material requirements
lot sizing X X
planning (MRP)
lot splitting X material safety data sheet
lot traceability X X
(MSDS)

16 ©APICS
CPIM Key Terminology
Part 1 Part 2 Part 1 Part 2
materials handling X on-time schedule
X
materials management X performance
mean X open order X X
mean absolute deviation operating expense X
X
(MAD) operation X
mean absolute percent error operational performance
X X
(MAPE) measurements
mean squared error (MSE) X operation costing X
mean time between failures operation due date X
X
(MTBF) operation duration X
mean time to repair operation overlapping X
X
(MTTR) operation/process yield X
measure phase X operations management X
median X operations plan X
milk run X operations scheduling X
min-max system X operations sequence X
mixed-flow scheduling X operations sequencing X
mixed-model production X X operation start date X
mixed-model scheduling X X operations strategy X
mix forecast X operation time X
mode X operator flexibility X
modular bill of material X opportunity cost X
modularization X option X
move card X option overplanning X
move time X X order entry X
moving average X ordering cost X
muda (waste) X X order picking X
multilevel bill of material X order point X
multilevel master schedule X order policy X
multisourcing X order priority X
mura X order promising X
muri X order qualifiers X
nesting X order release X
net present value X order winners X
net requirements X outbound stockpoint X
network planning X outlier X
nominal group technique X outsourcing X X
nonconformity X overall equipment
nonevident failure X X
effectiveness (OEE)
nongovernmental overhead X X
X
organization (NGO) overhead allocation X
non-value-added X overlapped schedule X X
normal distribution X overload X
obsolete inventory X overstated master
one-card kanban system X X
production schedule
one less at a time X owner's equity X
one-piece flow X pacemaker X X
on-hand balance X package to order X

CPIM Exam Content Manual 17


CPIM Key Terminology
Part 1 Part 2 Part 1 Part 2
pallet positions X priority control X X
panel consensus X priority planning X X
parent item X private carrier X
Pareto's law X X probability X
participative probability distribution X
X X
design/engineering probable scheduling X
participative management X problem-solving storyboard X
payback X process batch X
P:D ratio X process capability X
pegging X process capability index X
people involvement X process control X
perceived quality X process costing X
performance measure X process flexibility X X
performance measurement process flow X
X
system process flow analysis X
performance objectives X process flow diagram X
performance standard X process flow production X
periodic replenishment X process flow scheduling X
period order quantity X process focused X
perpetual inventory record X process manufacturing X
phantom bill of material X processor-dominated
physical inventory X X
scheduling
physical supply X process train X
picking list X procurement X
pickup and delivery costs X procurement lead time X
pipeline stock X product configuration
X
plan-do-check-action (PDCA) X X catalog
plan for every part (PFEP) X product cost X X
planned load X product differentiation X
planned order X product family X
planned order receipt X product focused X
planned order release X product group forecast X
planning bill of material X X production activity control
X X
planning horizon X X (PAC)
planning time fence X production capability X
point of sale (POS) X X production forecast X
point-of-use delivery X production level X
point-of-use inventory X production line X
poka-yoke (mistake-proof) X production plan X X
post-deduct inventory production planning X
X
transaction processing production rate X
postponement X X production schedule X
pre-deduct inventory production scheduling X
X
transaction processing productive capacity X X
prevention costs X productivity X X
preventive maintenance X X product layout X
primary work center X product life cycle X
priority X product line X

18 ©APICS
CPIM Key Terminology
Part 1 Part 2 Part 1 Part 2
product load profile X random variation X
product mix X X rate-based scheduling X
product-mix flexibility X rated capacity X X
product positioning X raw material X
product profiling X receiving X
product/service hierarchy X record accuracy X
profit margin X redundancy X
program evaluation and regression analysis X
X
review technique (PERT) released order X
project costing X remanufacturing X X
projected available balance X remedial maintenance X
project management X X reorder quantity X
project manufacturing X repair order X
project phase X repetitive manufacturing X X
project plan X replanning frequency X
protective capacity X replenishment lead time X
protective inventory X request for quote (RFQ) X
protective packaging X required capacity X
prototyping X requirements explosion X
pull signal X requisition X
pull system X X rescheduling X
purchase order X residual income X
purchase requisition X resiliency X
purchasing lead time X resource X
push system X X resource-constrained
X
pyramid forecasting X schedule
QS 9000 X resource leveling X
qualitative forecasting resource-limited scheduling X
X
techniques resource planning X X
quality X resource profile X
quality at the source X responsible procurement X
quality circle X return on investment (ROI) X
quality control X X reverse auction X
quality costs X X reverse logistics X
quality function deployment rework X
X X
(QFD) risk management X
quantitative forecasting risk pooling X
X
techniques robust design X
quantity discount X root cause analysis X X
queue X X rough-cut capacity planning
queue time X X
(RCCP)
quick asset ratio X routing X X
quick changeover X running sum of forecast
radio frequency X
X errors
identification (RFID) run time X X
random cause X safety capacity X
random-location storage X safety lead time X
random sample X safety stock X

CPIM Exam Content Manual 19


CPIM Key Terminology
Part 1 Part 2 Part 1 Part 2
sales and operations SMART X
X
planning (S&OP) smoothing constant X
sales plan X smoothing factor X
sales promotion X social responsibility X
sample X special cause X
sampling distribution X specification X
sawtooth diagram X specific identification X
scatter chart X X split lot X
scatterplot X spread X
scheduled downtime X standard X
scheduled load X standard costs X X
scheduled receipt X standard deviation X
scheduling X standard time X
scheduling rules X standardized work X
scrap X start date X
scrap factor X statistical process control
X X
seasonal index X (SPC)
seasonal inventory X statistical quality control
X
seasonality X (SQC)
second-order smoothing X stockkeeping unit (SKU) X
self-directed work team X stockout costs X
semifinished goods X stockout percentage X
sensei X store X
service X strategic drivers X
service function X strategic performance
X
service industry X measurements
service level agreement strategic plan X
X strategic planning X
(SLA)
service parts X strategic sourcing X
setup X subcontracting X
setup costs X substitution X
setup time X X summarized bill of material X
shelf life X sunk cost X
Shingo's seven wastes X X super bill of material X
shipping manifest X supermarket approach X
shitsuke X supplier X
shojinka X supplier certification X
shrinkage X supplier-input-process-
single-card kanban system X output-customer (SIPOC) X
single exponential diagram
X supplier lead time X
smoothing
single-level bill of material X supplier measurement X
single-minute exchange of supplier partnership X
X supplier relationship
die (SMED) X
single-source supplier X management (SRM)
six sigma X X supplier scheduling X
small group improvement supply chain X
X supply chain management X
activity

20 ©APICS
CPIM Key Terminology
Part 1 Part 2 Part 1 Part 2
surge capacity X transfer batch X
sustainability X X transfer pricing X
SWOT analysis X transient state X
synchronized production X transit inventory X
tactical plan(s) X transit time X
tactical planning X transportation X X
Taguchi methodology X transportation inventory X
takt time X X trend X
target inventory level X trend forecasting models X
tariff X truckload carriers X
terminals X two-bin inventory system X
terminal-handling charges X two-card kanban system X X
terms and conditions X two-level master schedule X
theoretical capacity X U-lines X
theory of constraints (TOC) X X uniform plant loading X
theory of constraints unit cost X
X
accounting UN Global Compact
X
third-party logistics (3PL) X X Management Model
throughput X X United Nations Global
X
throughput time X Compact
time-based competition unitization X
X
(TBC) unit load X
time bucket X unit of measure X
time buffer X unplanned repair X
time fence X upper control limit (UCL) X
time-phased order point upper specification limit
X X X
(TPOP) (USL)
time series X upstream X
time series analysis X usage variance X
time standard X utilization X X
tolerance X X value added X X
total cost curve X value analysis X
total cost of ownership value chain X
X
(TCO) value chain analysis X
total costs X value perspective X
total factor productivity X value stream X X
total line-haul cost X value stream mapping X
total productive variable cost X X
X X
maintenance (TPM) variable costing X
total quality control (TQC) X variance X
total quality management VATI Analysis X
X X
(TQM) velocity X
traceability X X vendor-managed inventory
tracking capacity strategy X X
(VMI)
tracking signal X vertical dependency X
trading partner X vertical integration X
traffic X virtual cell X
transaction channel X virtual organization X

CPIM Exam Content Manual 21


CPIM Key Terminology
Part 1 Part 2
visual control X
visual management X
visual review system X
voice of the customer
X X
(VOC)
wait time X
wall-to-wall inventory X
warehouse demand X
warehousing X
waste X X
wave picking X
waybill X
ways X
weighted moving average X
what-if analysis X
where-used list X
work cell X
work center X X
work in process (WIP) X X
work order X
workplace organization X
yield X X
zone X
zone picking X

22 ©APICS
systems. The fundamentals of planning,
APICS CPIM Part 1 priorities and capacity, execution, controls,
(previously known as Basics of Supply Chain
and performance measures are included.
Management)
The fourth section of the content outline is
Examination Committee
devoted to basic supply issues covering
inventory costs, functions, and metrics. It
Marilyn M. Helms, (Chair) D.B.A., CFPIM, CIRM,
provides a fundamental overview of supplier
CSCP, CMQ/OE
management, demand management, and
Dalton State College
monitoring supplier performance. Physical
Louise Beauchamp, CFPIM
distribution systems encompass
DBM Systems Inc.
transportation, warehousing, reverse logistics,
Staffan Cederstrand, CPIM
and distribution requirements planning.
EG Sverige AB
Lawrence Ettkin, Ph.D., CFPIM
The successful candidate will understand and
University of Tennessee at
be able to discuss the major management
Chattanooga
philosophies used in a supply chain. Emphasis
Dave Lehman, CFPIM, CIRM, CSCP, CLTD
is on manufacturing, but the examination also
DCL Associates, Inc.
covers the distribution, service, and retail
William S. Thurston, CPIM, CIRM, CSCP, PMP
industries. This understanding includes the
fundamental relationships in the design,
Scope of the Subject Matter
planning, execution, monitoring, and control
The entry-level concepts in this Exam Content that occur. The candidate should also
Manual contain essential information to understand:
prepare for the CPIM examinations.
 fundamental relationships among
The subject matter of CPIM Part 1 is an supply chain activities
introductory overview of CPIM Part 2, which
 basic enterprise resources planning
covers topics in greater depth.
 Introductory measurement and
The first section of the content outline covers continuous improvement.
basic business-wide concepts, including an
understanding of the various supply chain CPIM Part 1 Content Outline
environments. Common management The following table identifies the four main
concepts and techniques, including: supply topics of the exam. The relative importance of
chain fundamentals, operating environments, these topics varies among industries, but the
financial fundamentals, enterprise resources figures show the percentage designated for
planning, lean, quality fundamentals, and the each section of the exam.
theory of constraints.
Diagnostic Main topic Percentage of
The second section of the outline covers part exam
demand management, including a basic I Business-wide 25%
understanding of how markets influence Concepts
demand, how customers in these markets II Demand 25%
define value for goods and services and then Management
managing all demands to support the III Transformation 25%
marketplace. of Demand
into Supply
The third section of the outline covers IV Supply 25%
transformation of demand into supply and
includes the design of products (goods and
services), processes, and information

CPIM Part 1 Content 23


I. Business-wide Concepts core values in the areas of human
rights, labor standards, the
In this section, common management environment, and anticorruption.
concepts and techniques including: supply
chain fundamentals, operating environments, B. Operating Environments
financial fundamentals, enterprise resources
planning, lean, quality fundamentals, and Global, domestic, environmental, and
theory of constraints. stakeholder influences affect the key com-
petitive factors, customer needs, culture,
A. Supply Chain Fundamentals and philosophy of a company. This
environment becomes the framework in
The concept of a global network used to which business strategy is developed and
deliver products and services from raw implemented.
materials to end consumers through a
structured flow of information, physical 1. The definition and impact of the
distribution, and cash. It includes operating environment depends on
managing conflicts that occur within the customer expectations; cumulative lead
supply chain. Businesses are also called times, inventory, sustainability, product
upon to demonstrate social responsibility design, and life cycles.
in operating their supply chains.
2. Process choices for products and
1. A supply chain is a network of retailers, services include flow, intermittent, and
distributors, transporters, storage project.
facilities, and suppliers that participating
in the production, delivery, and sale of a 3. Production environment strategies
product or service to the consumer. It include engineer-to-order, make-to-
also includes returning items from the order, assemble-to-order, make-to-stock,
consumer back to the producer. and remanufacturing.

2. Supply chain conflicts and risks exist C. Financial Fundamentals


among trading partners that need to be
identified, analyzed, and addressed. Basic financial statements define the
Some examples include disruption of financial reporting common to most
supply, synchronizing supply with businesses. Underlying costs and analysis
demand, minimizing inventory terms provide further understanding of
investment, maximizing customer statement information and often serve as
service, and managing total cost. the basis for management decisions.

3. Internal organizational conflicts can 1. Balance sheets, income statement, and


exist between finance, sales, production, cash flow statement make up the
marketing, engineering, and planning standard financial reporting tools.
functions. Some examples include
excessive inventory versus inventory 2. Financial reporting must take into
stockouts; setup cost versus economies account the cost of goods sold, general
of scale; and expediting versus not and administrative costs, and fixed
expediting. versus variable costs.

4. The United Nations Global Compact 3. Financial data are used to analyze cash
addresses corporate sustainability in the flow, profit and loss, margin and
world economy by asking companies to throughput, inventory velocity, and the
embrace, support, and enact a set of

24 ©APICS
make-or-buy decision as it relates to incorporates: quality control tools, quality
total cost. costs, quality function deployment,
employee involvement and empowerment,
D. Enterprise Resources Planning (ERP) continuous process improvement, six
sigma, variation, process capability and
ERP is a framework for organizing, defining, control, and benchmarking.
and standardizing the business processes
necessary to effectively plan and control an G. Theory of Constraints (TOC)
organization so it can use its internal
knowledge to seek external advantage. The A philosophy that focuses the resources of an
objective for using ERP is the cross- organization on managing throughput and
functional integration of planning, financial performance. Key characteristics
executing, controlling, and measuring and techniques include product flow analysis;
functions required to effectively operate a throughput accounting; constraints
business organization to meet customer management; and continuous improvement.
expectations.
References: 1; 10 (chapters 1, 2, 6, 9, 14–
Key characteristics of ERP include its use 16); 13; 19; 20
as an integrated knowledge and decision-
making tool, cross-functional alignment Note: The first number indicates the sequence
within the organization and also with its number for the reference in the Bibliography
partners, the closed loop (feedback) section on pages 9–10 and the numbers in
mechanism, what-if simulation parenthesis indicate the relevant chapters
capabilities, and integrated financial data within that reference.
and performance measurement functions.
II. Demand Management
E. Lean
This section covers sources of demand for goods
Lean is a philosophy that emphasizes the and services, including a basic understanding of
minimization of the amount of all the markets, voice of the customer, and an overview
resources (including time) used in the of demand planning.
various activities of the enterprise.
A. Market Driven
1. Lean objectives are comprised of the
elimination of waste, providing value Consumer needs, competitive sources,
from the customer’s perspective, and economic conditions, and government
continuous improvement. regulations determine the demand
experienced by suppliers.
2. Key characteristics include flow
manufacturing, process flexibility, quality B. Voice of the Customer
at the source, supplier partnerships,
employee involvement, total productive Actual customer word descriptions of the
maintenance, pull systems, and work functions and features that customers desire
cells. for goods and services

F. Quality Fundamentals C. Demand Management

Quality management focuses on customer Demand management is the function of


needs using a variety of tools and recognizing all sources of demands for goods
techniques. The objective of quality and services to support the marketplace.
management is to increase profitability Demand management serves as a key input
and customer satisfaction. Quality

CPIM Part 1 Content 25


into the sales and operations plan and master 3. Information systems should follow
production schedule (MPS). product and process design. Data
governance is necessary to ensure data
1. Sources of independent demand that record accuracy.
must be considered are forecasts along
with customer, service, replenishment, B. Capacity Management
and inter-company orders. This section includes the function of
establishing, measuring, monitoring, and
2. Forecast management consists of adjusting limits or levels of capacity to
understanding the principles of execute all schedules. Capacity management
forecasting, the characteristics of encompasses resource requirements
demand, various forecasting techniques, planning, rough-cut capacity planning,
forecast error measurement, and capacity requirements planning, input/output
managing the variability of demand. controls, and constraints management.

3. Order processing occurs upon receipt of C. Planning


a customer’s order. Goods or services
will be fulfilled based on the operating Includes the process of setting goals for
environment. the organization and choosing how to use
the organization’s resources to achieve
References: 1; 10 (chapters 1, 8, 16) them. These different planning techniques
vary depending on traditional, lean, or
III. Transformation of Demand into Supply Theory of Constraints operating
environments.
This section includes the design of products
and services, capacity management, planning, 1. Strategic planning/hoshin planning
execution and control, and performance
measurements. 2. Business planning

A. Product and Process Design 3. Sales and operations planning,


production planning, and resource
Design affects product and process, the requirements planning
resulting framework of planning system
parameters, and the requirement for data 4. Master production scheduling and rough
appropriate in source, content, and cut capacity planning
accuracy. Collaboration with customers and
suppliers will improve product and process 5. Material requirements planning and
design. capacity requirements planning

1. Products and the processes used to 6. Final assembly scheduling and input/ output
make them are designed to create control
products more appealing to customers,
to improve productivity, competitiveness, 7. Advanced planning and scheduling
and sustainability.
8. Project management
2. Participative design/engineering ensures
that the final design meets all the needs
of the stakeholders and should ensure
products or services can be quickly
brought to the marketplace while
maximizing quality and minimizing costs.

26 ©APICS
D. Execution and Control 1. Inventories can be classified according to
their functions. This includes buffer,
The interrelationships between production transportation, lot size, anticipation,
activity control techniques (input/output fluctuation, hedge, as well as
control, kanban, constraints management) maintenance, repair, and operating
and planning schedules are synchronized to supplies (MRO).
meet customer service requirements.
2. Inventories are monitored using the
1. The output of material requirements following methods: ABC classification,
planning is used to execute the production physical inventory, cycle counting, record
plan and material releases. accuracy, days of supply, and inventory
turns.
2. Operations are executed using forward,
backward, finite, infinite, mixed model, 3. Inventories can be replenished using
kanban, drum-buffer-rope and constraint push or pull systems.
scheduling.
4. Inventory management decisions must
3. Techniques for maintaining and consider: item costs, carrying costs,
communicating shop floor order status ordering costs, stockout costs, and
include capacity control, production capacity-associated costs. These include
reporting, priority control, and flow control. strategies and policies related to
customer service and return on
4. An important part of execution and control investment.
is focusing on quality assurance by
measuring quality, monitoring process 5. Accounting uses four methods to value
variation, and improving process control. inventory: first in first out, last in first
out, average cost, and standard cost.
E. Performance Measurements
B. Purchasing Cycle
Key performance indicators are metrics
used to assess organizational The function and responsibility for
performance against strategic and tactical understanding demand, sourcing, procuring
goals. materials, supplies, or services, receiving
goods, and approving invoices for payment.
References: 1; 10 (chapters 1–7, 9–11, 14–
16) 1. The purchasing process begins with any
of the following signals: requisition,
IV. Supply material requirements planning (MRP)
output, kanban, and buffer.
This section includes the actual or planned
provision of a product, component, or service 2. Sourcing includes supplier selection,
and its sustainability. certification, agreements, and partnerships,
including vendor-managed inventory (VMI).
A. Inventory Total acquisition costs must be considered.

The stocks or items used to support 3. Order processing includes purchase


production (raw materials and work-in- release, defining terms and conditions,
process items), supporting activities and monitoring supplier performance.
(maintenance, repair, and operating
supplies), and customer service (finished 4. Order completion includes receipt of
goods and service parts). goods and approval of invoice.

CPIM Part 1 Content 27


C. Distribution 7. The total-cost concept takes the position
that all logistical decisions providing
The activities associated with the movement of equal service levels should favor the
material between the supplier, manufacturer, option that minimizes the total logistical
and customer. These activities encompass the costs across all areas versus a cost
functions of transportation, warehousing, reduction in a single area.
inventory control, material handling, order
administration, site and location analysis, References: 1; 10 (chapters 7, 9–13, 15)
industrial packaging, data processing, and the
communications network necessary for Key Terminology
effective management. It includes all activities An understanding of the list of terms on pages 11-
related to physical distribution, as well as the 22 of this document is strongly recommended.
return of goods to the manufacturer. The list is intended to be thorough but not
exhaustive. The candidate is also expected to be
1. Global distribution encompasses the familiar with the definitions of terms identified in
movement of goods around the world. the content outline. Definitions of these terms can
Decision factors include lead time, be found in the APICS Dictionary, 15th edition.
tariffs, and regulatory compliance.
Bibliography
2. Transportation is the function of All test candidates should familiarize
planning, scheduling, and controlling themselves with the references for this
activities related to mode, carrier type, examination. The recommended references
and movement of inventories across the pertaining to the diagnostic areas are listed at
supply chain. the end of each section of the content outline.
The first number indicates the sequence
3. Warehousing consists of the activities number for the reference in the Bibliography
related to receiving, storing, and section on pages 9-10 and the numbers in
shipping materials. parenthesis indicate the relevant chapters
within that reference. The text, Introduction to
4. Distribution inventory typically consists Materials Management, covers the majority of
of service parts and finished goods the material for the CPIM Part 1 exam. The
located in a distribution system. other references provide coverage of some of
the topic areas and can enhance candidates’
5. A distribution channel is the route, from understanding of the body of knowledge. The
raw materials through consumption, along APICS Dictionary is available from the APICS
which products travel. A transaction Book store; please visit apics.org/shopapics.
channel is a distribution network that Other references can be found at the APICS
deals with change of ownership of goods Amazon store at,
and services including the activities of astore.amazon.com/apics01-20.
negotiation, selling, and contracting.

6. Reverse logistics is a complete supply


chain dedicated to the reverse flow of
products and materials for the purpose
of returns, repair, remanufacture, or
recycling.

28 ©APICS
CPIM Part 1 Sample Questions 4. Which of the following statements about
The following ten multiple choice questions forecasting is true?
and four Technology Enhanced Item (TEI)
question examples are similar in format and (A) Forecasts are more accurate for
content to the questions on the exam. These individual products.
questions are intended for practice and to (B) Forecasts are most useful for items
illustrate the way questions are structured. with dependent demand.
The degree of success you have in answering (C) Forecasts should include an
these questions is not related to your potential estimate of error.
for success on the actual exam and should (D) Forecasts typically are more
not be interpreted as such. accurate when projected over a
longer period.
Read each question, select an answer, and
check your response with the explanation on 5. Intrinsic forecast data should be based
pages 50-51. on which of the following
considerations?
1. The shipping buffer in the drum-buffer-
rope scheduling process serves which of (A) Judgment, intuition, and informed
the following functions? opinions
(B) Economic indicators
(A) It is used to create the master (C) Shipment history
production schedule. (D) Sales history
(B) It provides protection for the order
due date. 6. Which of the following approaches
(C) It provides protection to the represents the longest planning range in
constraint. capacity management?
(D) It is used to release work to the
floor. (A) Capacity requirements plan
(B) Resource requirements plan
2. When using the 5S approach, which (C) Rough-cut capacity plan
step is implemented last? (D) Input/output control

(A) Sequence 7. An order of 10 components requires 16


(B) Sort standard hours. How much time should
(C) Straighten be allocated if the work center has an
(D) Self-discipline efficiency of 80% and a utilization of
80%?
3. Which of the following statements is
best used to understand a company's (A) 22.40 hours
ability to pay its bills? (B) 10.24 hours
(C) 16.00 hours
(A) Cash flow (D) 25.00 hours
(B) Income
(C) Balance sheet
(D) Market-share

CPIM Part 1 Content 29


8. The primary objective of a random- Technology Enhanced Items (TEI)
location storage system is to improve: Technology enhanced items are exam
question and answer sets that include
(A) distribution. specialized interactions for collecting exam
(B) kitting. question response data. Different types of
(C) access to stock. TEI’s include items with multiple responses
(D) use of space. (select all that apply), building lists / ordering
processes, fill in the blank, and drag and drop.
9. Which of the following types of carriers
lease or own their equipment, operate it The following examples were created to
themselves, and are typically company- familiarize exam candidates with
owned? technologically enhanced items that can be
found on the CPIM Part 1 exam.
(A) Common
(B) Contract Example 1: Fill in the Blank
(C) Private
(D) Parcel Average Current Retained Annual Cost
Inventory Inventory Earnings of Goods Sold
10. What type of inventory creates
independence between supply and the
use of material? $200,000 $400,000 $300,000 $800,000

(A) Cycle Given the data above, the inventory turnover


(B) Transit is _______ times. Please round answer to the
(C) Decoupling nearest whole number.
(D) Hedge

Example 2: Unordered List

Which of the following documents must be in


agreement before payment is made to the
supplier? Select all that apply.

 Bill of lading

 Customs documentation

 Invoice

 Packing list

 Purchase order

 Purchase requisition

 Receiving report

30 ©APICS
Example 3: Ordered List with Drag and Drop

Please use the drag and drop functionality to


answer the following question.

From the following options, please select the


sales and operations planning (S&OP) steps
involved in the monthly planning cycle and
place them in the correct order.

Example 4: Ordered List with Drag and Drop

Please use the drag and drop functionality to


answer the following question.

Place the 5S steps in the proper sequence for


implementation

CPIM Part 1 Content 31


Detailed Scheduling and Planning includes
APICS CPIM Part 2 principles and techniques of inventory
management, material requirements
Examination Committee
planning, capacity requirements planning, and
procurement and supplier planning.
Andrea M. Prud’homme (Chair), Ph.D.,
Recognizing the importance of supply chain
CFPIM, CIRM, CSCP, CLTD
management, this subject area also covers
The Ohio State University
deployment of supply chain strategies related
Al Bukey, CFPIM, CIRM, CSCP, SCOR-P
to scheduling, planning, and sourcing. This
Abcan Consulting
supports the strategies and objectives
Carol M. Bulfer, CPIM
established by the company, as constrained
Parker Hannifin Corporation
by lead time, cost, equipment, personnel,
Matt Drake, Ph.D., CFPIM, CLTD
sustainability considerations, or other
Duquesne University
constraints. This encompasses requirements
Murray Olsen, CFPIM, CIRM, CQA, CMQ/OE,
to bridge the master planning with the
CSSGB
execution and control subject areas.
Northrop Grumman
Michael A. Roman, CPIM
Execution and Control of Operations
Manufacturing Practices, Inc.
encompasses the principles and techniques
Jaap Stumphius, CFPIM, CLTD
necessary to execute, control, and manage
Telio Nederland
operations, and participate in design
Rosemary Van Treeck, CPIM, CIRM, CSCP
decisions. This subject area provides
Gateway Technical College feedback about how good plans are being
executed, as well as providing information for
Scope of the Subject Matter customers and suppliers about the status of
Please read the introductory material in this services and products in process. The
manual for essential information pertaining to importance and emphasis of these
the examination. This exam includes the approaches are relative to the operation’s
following four major subject areas: Master environment, the labor environment, the
Planning of Resources, Detailed Scheduling physical organization of the facility, and the
and Planning, Execution and Control of effectiveness of management and employees.
Operation, and Strategic Management of
Resources. The concepts included in this Strategic Management of Resources includes
subject area apply to manufacturing and higher-level thinking on strategic planning and
service organizations. implementation of operations. This requires
an understanding of how market requirements
Master Planning of Resources includes and strategic position of the organization drive
principles and techniques of demand the resources and processes of an
management, sales and operations planning, organization. This includes how operational
master scheduling, and distribution planning. strategies are developed and implemented,
It includes forecasting, order servicing, change management and risk implications.
demand shaping, customer relationship
management, and distribution network and The successful candidate will have a thorough
replenishment. Also included is the integration understanding and be able to apply all key
between the sales and operations plan, terms, principles and techniques contained
production plan, and master production within the four subject areas, analyze
schedule. situations to determine which approaches are
applicable, and recognize when to escalate
issues.

32 ©APICS
CPIM Part 2 Content Outline methods of forecasting, fitting a forecast
The following table identifies the four main to the situation, and tracking its
topics of the exam and shows the percentage performance over time. Techniques
designated for each section of the exam. include:

Diagnostic Main topic Percentage a. The application of basic qualitative


part of exam and quantitative techniques
I Master Planning of 25% b. Evaluation of forecasting
Resources performance
II Detailed Scheduling 25% c. Collaboration with customers and
and Planning suppliers to improve forecast
III Execution and 25% accuracy
Control of
Operations 3. Management of the customer interface:
IV Strategic 25% Effective management of the customer
Management of interface involves making realistic order
Resources delivery promises and maintaining
positive customer relationships.
Principles and techniques include:
I. Master Planning of Resources
a. Customer relationship management
A. Demand Management (CRM) concepts
b. Setting customer service policies,
Strategic and business planning is closely safety stock, and performance
related to the management of demand, targets
including forecasting and managing the c. Maintaining effective customer
customer interface. communications
d. Techniques for measuring order
1. Principles of demand management: delivery performance
Successful planning requires an e. Influencing demand to better align
understanding of how components, with supply
concepts, and linkages of the strategic
plan, the business plan, and the master References: 1; 4; 16 (chapters 3-4)
planning of resources interact.
Principles include: Note: The first number indicates the sequence
number for the reference in the Bibliography
a. The planning hierarchy section on pages 9–10 and the numbers in
b. Relating the planning process to parenthesis indicate the relevant chapters
manufacturing and service within that reference.
environments
c. Demand forecasting concepts B. Sales and Operations Planning (S&OP)
d. The relationship between the
purpose of the forecast and required Sales and operations planning (S&OP) concepts
timeliness, and accuracy of the data and techniques are used to link strategic goals to
e. Management considerations related operations and coordinate the various planning
to forecast method selection efforts of the functional areas, including
f. Selecting a time horizon and operations, sales, sourcing, product
interval, and a level of aggregation development, marketing, and finance in a variety
of business environments.
2. Techniques of forecasting demand:
Effective demand forecasting requires
knowledge of the uses of a forecast,

CPIM Part 2 Content 33


1. Principles of sales and operations e. Developing and validating a
planning: S&OP is the process executive production plan in a variety of
management uses to control and guide production environments
the business. It coordinates the various f. Executing resource planning
business functions to ensure they g. Assessing the desirability of
support the overall organizational alternative production plans
strategy and enhance customer service.
It integrates the multiple plans References: 1; 4; 16 (chapters 5, 10)
developed within a business and
provides direction for more detailed C. Master Scheduling
short-term to medium-term planning.
Principles include: Master scheduling is the process of translating
higher-level aggregate plans into feasible
a. The role of S&OP in the planning and schedules that operations and suppliers can
control hierarchy execute.
b. Key linkages to the S&OP process
c. Different methods for balancing 1. Principles of master scheduling: The
supply and demand master scheduling process
d. Trade-offs related to different disaggregates the production plan into
volume/mix combinations an executable schedule that links
e. The impact of different business shipment of customer orders to
environments including material management. A master
manufacturing and services on the schedule must be developed that
S&OP process promotes operational stability and good
f. Evaluating alternative plans and customer service while maintaining
associated risks realistic priorities. Principles include:
g. Assessing the financial implications
of the plan a. The role of master scheduling in the
h. Identifying stakeholders and their planning and control hierarchy
responsibilities in the process b. Linkages with other planning
i. Identifying the planning horizon and processes within the planning and
the appropriate aggregation level control hierarchy
j. Trade-offs between customer service c. Relationships between master
levels and inventory or backlog scheduling, capacity management,
levels and materials management
d. The impact of different business
2. Techniques of sales and operations environments including manufacturing
planning: There are several steps and services on the master scheduling
involved in the process of establishing a process
sales and operations plan. A key output e. The relationship between the
of S&OP is the production plan. manufacturing environment and
Techniques include: product structure
f. Determining the level in the bill of
a. The various levels of management material where the master schedule
involved and their roles should be developed
b. The sequential steps of the S&OP
process and their interactions 2. Techniques of the master scheduling
c. The various inputs and outputs of process: The process of developing a
each of the S&OP steps master schedule involves numerous
d. Performance measures of the S&OP decisions related to the mechanics and
process the selection of techniques and tools.

34 ©APICS
Effective master scheduling requires D. Distribution Planning
significant insights into each of these
choices. Techniques include: Distribution planning covers the planning of
distribution networks and replenishments in
a. Identifying and quantifying sources the distribution environment. It also covers
of demand to be considered in the concepts and techniques used to plan the
master scheduling process location of distribution centers, the
b. The mechanics of creating, transportation logistics between supply sources
maintaining, and using the master and outlying locations, and the replenishment
schedule of these locations in a variety of business
c. Reviewing capacity requirements environments.
and the role of rough-cut capacity
planning (RCCP) 1. Principles of distribution planning:
d. Developing and using a final Methods and elements of logistics
assembly schedule (FAS) planning are used to establish the
e. Measuring performance of the supply channel locations and
master scheduling process transportation schemes. Principles
f. The roles and responsibilities of the include:
master scheduler
g. Planning and coordinating changes a. Various distribution strategies and
in inventory levels, backlog, capacity, network configurations
major customer orders, time fences, b. Transportation planning, controlling
product and process designs, and and scheduling activities
suppliers c. Inventory levels and locations
h. Maintaining the integrity of the required within the distribution
master schedule channel
i. Dealing with the consequences of an d. Risk management and sustainability
unrealistic master schedule considerations
j. Measuring actual performance
against the master schedule 2. Techniques of distribution planning:
Various approaches are used to
3. Advanced planning and scheduling evaluate performance of the
(APS): APS systems employ advanced distribution network and planning
computer capabilities and algorithms to process. Techniques include:
perform many of the functions related
to master planning of resources in real a. Developing a distribution location-
time. Principles and techniques specific product forecast
include: b. Developing the replenishment
planning parameters for each
a. Major components of an APS system stockkeeping unit (SKU) within the
b. Impact of APS techniques on master distribution network
production schedules c. Methods of time-phased planning
c. APS techniques based on logic inherent in distribution
simulation, optimization, or requirements planning (DRP)
constraints management d. Methodologies for linking DRP to the
approaches S&OP and master scheduling
processes
References: 1; 4; 16 (chapters 7, 10) e. Supporting specific marketing
efforts and managing supply
shortages
f. Pull systems

CPIM Part 2 Content 35


g. Push systems e. Lean concepts that improve
h. Data collection methods for effective throughput and reduce or eliminate
feedback inventories, including value-stream
i. Key distribution performance mapping techniques and pull
indicators systems
j. Feedback on distribution network f. The impact of global sourcing risks,
design decisions, distribution planning such as financial, political,
process improvements, and execution transportation, and environmental,
of the plan on inventory planning decisions

References: 1; 4; 8 (chapters 7-15); 16 2. Techniques of Inventory Management:


(chapters 14–16) Effective inventory management
includes understanding and applying the
II. Detailed Scheduling and Planning appropriate techniques for the different
types of inventory, business
A. Inventory Management environment and desired objectives of
inventory investment and customer
Inventory management principles, policies, service. Techniques include:
and techniques impact many other decisions
throughout the organization, including a. Order review methodologies,
stocking levels, order quantities, safety including material requirements
stocks, handling and storage requirements, planning (MRP), reorder point (ROP),
and financial management. periodic review, visual review, and
kanban/pull system triggers
1. Principles of Inventory Management: b. Lot-sizing techniques for
Inventory management policies are put in independent and dependent
place to meet both operational and demand applied to different types of
financial objectives in the organization. production and service
Principles include: environments in support of inventory
investment strategies
a. Types and classifications of inventory, c. Safety stock and safety lead time
including raw materials, work in techniques and their effects on inventory
process (WIP), finished goods, and customer service objectives
maintenance, repair, and operating d. Inventory valuation methods, including
(MRO) supplies, returned goods, FIFO, LIFO, transfer pricing, standard
excess, inactive, obsolete, scrap, and and actual cost, and project and
distressed and perishable inventory process cost
b. Trade-offs in stocking levels, e. Cycle counting, physical inventory,
customer service, environmental and ABC classification methods for
impact, and inventory accuracy improving and sustaining part count
targets in different business and inventory investment accuracy
environments f. Storage, flow and material handling
c. Inventory performance measures as options, including stock location
they relate to business objectives, systems, automated storage/
such as inventory turnover, retrieval systems (AS/RS), vendor-
customer service levels, and managed inventory (VMI), and
inventory accuracy consignment, as well as proper
d. Differences between dependent and transaction management
independent demand as related to g. The importance of proper
various inventory models and identification, country of origin
sources of demand declaration, documentation,

36 ©APICS
tracking, and traceability of d. Making decisions to facilitate
inventory movement including bar material planning, establish
coding and radio frequency priorities, resolve conflicts through
identification tagging pegging relationships, and support
other decisions and productivity
References: 1; 2; 7 (chapters 1, 2, 4–6, 8– measures based on the type of
10); 9; 10 (chapters 9–12); 14 (chapters 1, 2, environment and product life cycles
8, 9, 15); 15 (chapter 4); 16 (chapters 8, 9, e. Monitoring system feedback
13–14, 16); mechanisms to enable the appropriate
actions necessary to balance supply and
B. Planning Material Requirements to Support demand
the Master Schedule f. Using various safety policies to
minimize the impact of uncertainty
Planning material requirements driven by the on the planning process
master production schedule (MPS), including g. Integration with master planning,
material requirements planning (MRP), deals final assembly, and configuration
with dependent demand parts and processes to ensure material
interrelationships that require planning at any availability matches demand
given time. It also includes independent quantities, timing, and priorities
demand planning for service parts, matching
supply with demand, and managing demand 2. Techniques of the detailed material
at aggregate and disaggregate levels. requirements planning (MRP) process:
Demand for dependent components is
1. Principles of the material planning calculated by exploding bills of material
process: The inputs used in calculating (BOM), netting requirements, and
requirements for inventory items, determining time-phased inventory
including accuracy, timeliness and needs based on inventory policies.
completeness, are critical to the The output from the planning process
detailed material planning process. recommends the placement and
Principles include: replanning of supply orders to maintain
the validity of order priorities.
a. Defining inventory data, including item Techniques include:
description, current usage rates, stock
balances, and tracking historical a. Generating time-phased
demand as required to support the requirements to create material
material planning process. plans that support company and
b. Using master schedule data to supplier needs for long range and
describe types, quantities, sources, short range planning
priorities, customer orders, b. Using the MRP time-phased grid to
forecasts, and time phasing of display gross requirements, scheduled
product demand generated as a receipts, projected available, net
result of the master planning requirements, planned order receipts
process and planned order releases
c. Defining engineering data for c. System replanning of order priorities
product structure and to respond to changes of demand and
parent/component relationships, supply and resynchronize the material
including information on part plan with the current material
interdependencies, lead times, and requirements
effective management of d. Revising planning parameters to
engineering changes adjust lead time, lot size, safety stock
quantity, kanban quantity, cycle times,

CPIM Part 2 Content 37


and other parameters to reflect Principles include:
product life cycles, current conditions, a. Evaluating the availability of
and company strategy theoretical, demonstrated, and rated
e. Utilizing what-if analysis and capacity, qualified by efficiency and
simulation tools, methodologies, and utilization factors in different types
techniques to evaluate viable of environments
alternatives without changing the b. Calculating load from planned and
existing material plan released orders, repetitive schedules,
past-due orders, rework, and work in
3. Managing the project plan: Planning process
material requirements for specific project c. Assessing impacts of industry-
activities requires the implementation of specific conditions, such as
additional tools. Principles and remanufacturing, byproducts, co-
techniques include: products, and recycled material
d. Identifying the impact of unplanned
a. A project planning process that work and quality problems on
includes a statement of work, work resources
breakdown structure, project e. Scheduling, queuing, and managing
schedule, responsibility matrix, throughput in various industries,
resource requirements, and budget including service
b. A project implementation phase that
includes selecting the project team, 2. Techniques of the detailed capacity
scheduling the work, and managing planning process: Various approaches
the team dynamics, schedule, and can be used for applying the process
budget within different production and service
environments. The output of detailed
References: 1; 2; 7 (chapters 1, 14) 10 capacity planning is used to adjust the
(chapters 4, 11); 16 (chapters 7–9); 18 levels of capacity or load to complete
(chapters 3–4, 11–13, 17, 22) scheduled work within the required
timeframe. Techniques include:
C. Planning Operations to Support the Priority
Plan a. System design techniques including
infinite and finite capacity planning,
Capacity management encompasses queuing and sequencing, constraint-
planning, establishing, measuring, monitoring, based finite scheduling, load
and adjusting levels of capacity to execute the balancing for scheduling
master schedule and related materials plan. It manufacturing operations, and
addresses the balancing of the material plans establishing projected load on
with available internal and external resources manufacturing resources
and supporting activities, including constraint b. Simulation and modeling techniques
management, line and flow balancing, and that enable a variety of scheduling
variability and capacity in a transient state. and loading conditions for
transformation processes
1. Principles of the detailed capacity c. Loading operations and adjusting for
planning process: Work center and capacity reductions or increases, to
routing data are used to schedule the support business and customer
projected load from the material service targets and to accommodate
requirements plan by time period. process variability
d. Recognizing intangible and variable
capacity characteristics to manage
load in a service industry

38 ©APICS
e. Integration with master planning, of supply chain strategies to actual
final assembly, shop floor activities, sourcing techniques and the principle
and configuration processes to of total procurement costs. Principles
ensure that capacity availability include:
matches demand
f. Using safety capacity in a. Varying degrees of partnership,
environments that must including strategic alliance, joint
accommodate unplanned load venture, retailer-supplier, distributor
variability integration, contract manufacturing,
g. Various methods of balancing and technical and operational
capacity and load including partnering
rescheduling or splitting orders, b. Supplier selection alternatives
outsourcing, workforce including single or multiple sourcing,
development, changing capacity domestic and foreign providers, and
through workforce changes, and special services
modifying order quantities and c. The inclusion of additional supply
priorities chain links; including retail,
h. Managing constraints and balancing distribution, and transportation
flow using process flow scheduling companies
in process industries, in either batch d. Environmentally responsible
or continuous mode purchasing to minimize the impact
i. Measuring the capacity planning the organization and its suppliers
process on the ability to balance have on the environment
workload with available capacity and e. The effect of procurement planning,
support on-time performance new product introduction, and
engineering change control on
References: 1; 2; 10 (chapters 5–6, 15); 16 supply chain performance
(chapters 10–12)
2. Techniques for supplier partnerships:
D. Planning Procurement and External Elements of supplier relationships can
Sources of Supply vary, based on supplier relationship
management (SRM) principles and
Procurement and supplier planning activities, as well as different
encompasses planning and evaluation competitive environments and markets.
activities to qualify suppliers and establish Techniques include:
effective communication channels. Ongoing
relationships require defining mutual business a. Methods of procuring materials and
needs, sharing of product and process services including contracts,
information, the impact of global sourcing, kanbans, blanket orders, purchase
sustainability practices, and quality orders, consignment, pricing
management. This includes approaches to agreements, outsourcing and e-
communicating forecasts, orders, and commerce
schedules to ensure that purchased capacity b. Delivery approaches including
and materials will be available in the required traditional modes of transportation,
quantities and at the required time. third-party logistics (3PL), cross-
docking, point-of-use delivery, direct
1. Principles of supplier relationships: shipment and vendor-managed
Establishing long-term supplier inventory (VMI).
partnerships is necessary to ensure c. Supplier participation in product
competitive advantage and continuous design, quality requirements, related
improvement, including the application technology, sustainable business

CPIM Part 2 Content 39


practices, delivery, and accounting executing activities is necessary for
processes effective operations management.
d. Defining goals and benefits of the These principles are influenced by the
partnership, including improvements policies that determine how work will
in technology, inventory levels, be accomplished, such as push versus
customer service, quality, lead pull environments. Principles include:
times, visibility, value-chain
forecasting, cost, impacts on the a. Flow principles, including locating
environment, damage and loss bottlenecks, constraints management,
prevention, continuous minimizing waste, and coupling and
improvement, access to new decoupling operations
markets, and time to market b. Batch principles that employ the
e. Applying a supplier rating system to concepts of process batch, transfer
encompass quantitative measures, batch, lot size, and reorder point
such as cost and on-time delivery, (ROP)
product quality, environmental c. Scheduling principles that commit
impact, and qualitative measures resources to a timetable, including.
such as social performance, expediting, resolving imbalances,
workforce diversity, human rights, time fence policies, lead time
labor, and anticorruption components, schedule composition,
f. Understanding effective resource constraints, final assembly
communication techniques, cultural scheduling (FAS), and preventive
differences, commercial versus maintenance
government interests, and d. Integrating pull systems with
information technology material requirements planning
g. Using data necessary for (MRP) and enterprise resources
collaboration including risk sharing, planning (ERP) systems to align
technical and quality specs, shop floor activity with customer
engineering changes, supply chain demand and takt time
inventories, and future demand
2. Techniques of executing operations: A
References: 1; 2; 7 (chapters 3, 6–11, 14); 9; variety of techniques are available to
10 (chapters 7, 13, 16); 14 (chapter 15); 15 guide the execution of operations
(chapters 3, 5–9); 18 (chapter 8) plans. It is important to understand the
distinctions and applicability of using
III. Execution and Control of Operations different techniques based on the
manufacturing or service environment.
A. Execution of Operations Techniques include:
The execution of operations includes a. In a push environment, making
understanding the scheduling processes that decisions on sequencing rules for
translate plans into operational activities. This work and the impact on setup time
includes applying methods of authorizing and and changeovers, the movement of
releasing work, and the management of materials, the use of alternate
resources required to accomplish the work. All routings, forward and backward
execution activities rely on an understanding scheduling, load leveling,
of the decisions made about the input/output control (I/O), and
organizational environment, strategies, and adjusting constrained resources
objectives. b. In a pull environment, allowing work
to flow without interruption by using
1. Principles of executing operations: An mixed-model scheduling, rate-based
understanding of the key principles of

40 ©APICS
scheduling, pulling work to fulfill evaluate variances to standard
internal or external customer performance, and determine
demand, synchronization, line performance process stability,
balancing, and adjustments to process capability, and theoretical
constrained resources and demonstrated capacity
c. Elimination of non-value-added e. Managing the control process
activities to reduce the time to through established standards and
process goods and services, procedures in support of the
lessening cost to the organization organization’s goals and bottom line,
and improving competitive including the use of relevant and
advantage measurable performance measures.
d. Using the five S methodology to f. Using safety and environmental
organize, clean, develop, and standards to control and protect the
sustain a productive work organization and its environment.
environment
2. Techniques: The objective of control is
References: 1; 3; 10 (chapters 6, 12, 15); 11 to compare actual to planned results.
(chapters 6, 14, 17, 19) 13; 16 (chapters 10, Tools and sources of data vary
11, 13, 16, 17) depending on the environment.
Techniques include:
B. Control of Operations
a. Using costing methods, including job
Control of operations encompasses the ongoing costing, absorption costing, activity-
review and management of operational results based costing (ABC), and process
in comparison to the established near-term costing to compare actual to
plan, followed by analysis and application of any planned, budgeted or standard costs
corrective action required to align performance b. Capturing variances in cost through
with the plan. Control includes the principles inventory valuation, obsolescence
and techniques to accomplish the plan using review, scrap, rework, repairs,
both internal and external resources. returns, and defective output
c. Measuring costs related to quality
1. Principles of controlling operations: The including internal and external
principles of operational control focus on failure costs, as well as appraisal
the organization’s goals, feedback from the and prevention costs
process, and management of the control d. Evaluating and recording levels of
process. An understanding of the process activity to determine
operation’s environment and the variation and conformance to
distinction between internal and external standards by using statistical
elements is essential in establishing process control (SPC) methods such
appropriate measures. Principles include: as P charts, X-bar charts, R charts,
and capability indices
a. Expressing the detailed operational e. Evaluating throughput by measuring
plan in units, hours, labor, and efficiency, utilization, productivity,
inventory objectives takt time, cycle time, visual signals,
b. Analysis and review of the capability, and units of throughput such as
stability, and permissible tolerance quantities of materials, services,
flexibility levels of operations hours, or dollars
c. Applying control goals of product and
service design and vendor certification References: 1; 3; 10 (chapters 6, 9, 16); 11
d. Using feedback to provide data (chapters 5, 6, 7, 14, 16, 17, 18, 20, 21, 22,
necessary to maintain valid plans, 24, 25) 13; 16 (chapters 10, 17)

CPIM Part 2 Content 41


C. Quality, Communication, and Continuous b. The define, measure, analyze,
Improvement improve, control technique (DMAIC)
as an organized six sigma
Management defines the quality and continuous procedure for close examination of
improvement goals of the organization, and uses a process
meaningful communication techniques to educate c. The Shewhart/Deming cycle, also
others and bring about those goals. known as plan-do-check-action
(PDCA), which moves from planning
1. Principles of quality, communication and through the observation or data
continuous improvement: Management gathering, validation, and execution
principles address the organization, phases of the problem-solving
training, and motivation of work teams process
and other groups of persons assigned d. A3 problem solving including root
purposeful activities. Understanding that cause analysis, current and target
there are a number of different and often conditions, implementation, and
conflicting goals within the organization follow-up plans to ensure an
requires considering a broad range of objective has been met
possible solutions. Principles include: e. Considering the impact of
management style and contingency
a. Group principles, including multiple- planning on communication,
criteria decision making, consensus- problem solving, and decision-
building, gatekeeping, problem- making
solving processes, meeting skills,
understanding the capabilities and References: 1; 3; 11(chapters 2, 4, 5, 6, 7,
work of others, engaging in cross- 14, 15, 16, 19, 20, 21, 24, 25); 13
training, and gaining skills in effective
group formation and leadership D. Design Concepts and Trade-Offs
b. Individual principles including task
definition and guidance, managing Successful execution system design balances
“up” and “down” in the organization, competing organizational objectives. Effective
self-assessment, motivation of self execution system design will incorporate the
and others, understanding of work appropriate flow approach, customer definition
elements, asking diagnostic of quality, market demand for speed, use of
questions, and utilizing time available technologies, and the appropriate
appropriately workforce training, to simultaneously achieve
desired system outcomes, profitability, and
2. Techniques of quality, communication sustainability.
and continuous improvement:
Techniques apply the principles and No single operational design is appropriate for
include thoughtful problem definition all organizations or standard industry
and its impacts, criteria for evaluating classifications. The operational design must
solutions, and choice of the most be adapted to deliver competitive
effective solution given the criteria. manufacturing or service capability to the
Techniques include: marketplace. As an organization’s operations
personnel may be asked for input into new or
a. Basic tools of quality management changing design systems; they must
including check sheets, cause-and- understand general principles and techniques
effect analysis, Pareto charting and and tradeoffs of the underlying product,
analysis, the use of histograms, organization, and process designs.
scatter charts, control charting, and
flow charting

42 ©APICS
1. Principles of design concepts and tradeoffs: 2. Techniques of design concepts and
Design principles are classified as product tradeoffs: Design techniques consist of
design, organizational or plant layout, and the processes a manager might invoke
process design, and apply at the local, to arrive at an appropriate trade-off
network, and global level. Principles include: given stated objectives. Product design
techniques that influence operations
a. Local design principles that pertain include engineering methods that
to internal organizational processes, increase flexibility of service delivery.
and the selection of the appropriate Process design techniques can include
engineering approach which impacts the use of technology and lean
the execution system flexibility, principles. Techniques include:
process choice, and speed to
market a. Continuous improvement design
b. Making local process design trade- activities that enhance the internal
offs and improvements in a portion performance and relationships with
of the workspace, a department, or upstream and downstream
entire facility, including application customers and suppliers, including
of lean principles, quality systems, collaboration technologies,
automation, use of technology, incorporation of the voice of the
environmental footprint, cost- customer (VOC), and supplier
volume-profit relationships, and feedback
other tools that affect process b. Work area design activities including
outcomes setting specifications, work
c. Shaping the relationship with up and orientations, flows, layouts,
down stream customers and changeovers, simulation, modeling,
suppliers through network shop-based technology
principles, including the use of improvements, automation, and
collaboration technologies such as employee training
voice of the customer (VOC), supply c. Using collaboration and visual
chain management, logistics techniques in local work and
practices, lean tools, and quality employee groups to improve
principles performance in the workspace,
d. Forming the basis of an process, and information flow
organization’s relationship to the d. Utilizing quality systems and tools
world at large through global including quality function
principles, including costs and deployment (QFD), concurrent
benefits to society, green and engineering, modular design, and
sustainable manufacturing, and feature postponement to make
other corporate social execution systems more likely to
responsibilities produce desired outcomes
e. Understanding the tradeoffs and e. Using design of experiments (DOE)
compromises within the to measure controllable and
relationships of variables and design uncontrollable process variables and
parameters that often compete with their effects on managing
each other, such as the level of satisfactory outputs from the
product or service customization the process or system under analysis
execution system will be responsible
for delivering References: 1; 3; 10 (chapters 7, 14); 11
(chapters 4, 5, 6, 11, 14, 15, 17, 20, 22, 23)
13; 16 (chapters 13, 15, 17)

CPIM Part 2 Content 43


IV. Strategic Management of Resources b. Development of a business strategy
c. Development and alignment of
A. Understanding the Business Environment functional strategies
and Developing Corporate Strategy d. Development and alignment of
operating strategies
Operations strategy must be integrated with
the corporate strategy of the firm, reflecting the 3. Sustainability: Everyone within an
external environment, as well as organizational organization must understand the
priorities and philosophies. The operations importance and impacts of the three
strategy development process must capture parts of sustainability (economic,
and assimilate that information to provide environmental, and social, also known
context for alignment with corporate strategy. as the triple bottom line (TBL) to the
Corporate strategy development must consider organization, stakeholders, and the
business environmental factors. supply chain. Sustainability must be
purposely integrated with corporate
1. Environmental analysis: An analysis of strategy. Principles include the
the strategic environment, structure, implication of sustainability on:
and infrastructure will enable
organizations to more effectively align a. Product and process design
resources with the strategic plan to b. Supply chain design, including
create a competitive advantage. reverse logistics
Understanding the broad c. Waste hierarchy and circular
environmental content in which a economy implications
company’s industry is situated is d. Sustainability risk identification and
required. Principles and techniques mitigation
include: e. Sustainability metric identification,
reporting, and verification
a. Knowledge of the external
environment References: 1; 6 (chapters 3–9); 15
b. Performing an industry analysis (five-
force model) B. Developing the Operations Strategy
c. Performing an internal analysis (core
competencies) The operations strategy development process
d. Performing a value chain analysis by must align with the business strategy and
identifying the primary activities and reflect the analysis of the business
related support activities that create environment. This includes the processes of
customer value identifying, evaluating, and choosing among
e. Performing a situation analysis alternatives in the context of the business
(SWOT) to assess strengths and strategy, and understanding the significance
weaknesses, market opportunities, of the various factors in developing the
and external threats to future well- operations strategy.
being
1. Performance objectives/goals:
2. Strategy formulation: In order to Choosing how performance is
formulate an operations strategy, the measured and how success is
organization must identify appropriate determined should be considered
strategic components and understand during the development of the
the alignment of and relationships operations strategy.
between operations with corporate
strategic objectives. Principles include: 2. Product/service design: In order to
determine the strategy for product or
a. Development of a corporate strategy

44 ©APICS
service design, the firm’s capabilities organization, such as sales forecast,
and processes must be matched with capacity plans, and product planning
market needs and product life cycles. d. Planning human resource and
Principles include: technology requirements

a. A knowledge of competitive priorities 5. Measurement: Measurements must be


b. Determination of the firm’s order carefully chosen to support
winners and qualifiers performance objectives, including how
c. Production environment strategies they will drive human action and
including: engineer-to-order, make- behaviors. Measurement techniques
to-order, assemble-to-order, include:
configure-to-order, make-to-stock,
and remanufacturing a. Financial statement analysis to
appraise a company’s past financial
3. Structure and infrastructure: Making performance and its future potential
structure (capital-related) and by analyzing financial statements,
infrastructure (organizational) choices including horizontal analysis, vertical
is a key step in developing the analysis and ratio analysis
operations strategy. This includes b. Cash flow results from cash receipts
understanding that all decision areas and cash payments due to
have both structure and infrastructure operating, investing and financial
implications. Principles and techniques activities
include: c. Capital investment due to
commitment of resources to new
a. A paradigm or philosophy to provide product development, computer
a consistent framework for decision- systems, or a project that results in
making using tools such as lean, benefits spread over several time
theory of constraints (TOC) and periods, including obtaining new
enterprise resource planning (ERP) facilities, expansion or replacement
b. Process design such as the product- of existing facilities such as property,
process matrix and the product plant and equipment
service design matrix d. Costing methods that determine the
c. Vertical and horizontal integration cost of producing a product,
alternatives including absorption, variable,
activity-based costing (ABC) and job
4. Coordination strategy: During the order costing
development of an organizational e. Nonfinancial concepts, such as
strategy, consideration must be given balanced scorecard and operational
to aligning internal operations and and process measures
external supply chain partners. f. Cost-volume profit analysis of how
Principles and techniques include: profit and costs change with
changes in volume, including break-
a. Planning for capacity, such as lead even analysis, target income
and lag volume, and sales mix analysis
b. Integrating materials management
strategies across the supply chain, 6. Logistics: An understanding of logistics
including inventory levels, location, includes the principles and techniques
and ownership of forward and reverse supply chain
c. Sharing key information design, asset ownership, and
appropriately throughout the outsourcing options such as third- and
fourth-party logistics (3PL and 4PL)

CPIM Part 2 Content 45


7. Organizational commitment process: In a. Management involvement in the
order to gain broad acceptance of the organizational structure,
operations strategy from management measurement systems, and human
and the rest of the organization, it is resource policies and practices
necessary to understand any issues that b. Analysis of value creation, including
may prevent a commitment of resources product/service and
production/delivery
8. Risk assessment: It is critical to c. Implementation of quality strategies
understand the sources and and methodologies
probabilities of risk, impacts of d. The use of information through
potential failures, and the risk technology, visual management, and
tolerance of the organization, as well sharing processes
as prevention, recovery, and resiliency
strategies in developing the 4. Structural deployment: An organization
operations strategy must identify and implement any
required modifications to existing
References: 1; 5 (chapters 1–5); 6 (chapters operational structures to be consistent
2, 10, 12); 12; 15; 17 with the operations strategy. Principles
and techniques include:
C. Implementing the Operations Strategy
a. Aligning the facilities strategy
Various strategic leadership activities, b. Positioning equipment to support
leadership roles, and responsibilities are process flow
necessary for the successful implementation c. Identification and management of
of the operations strategy. assets, such as inventory and
vehicles.
1. Risk management: In order to mitigate
existing and changing risks, actions are References: 1; 6 (chapter 11); 12; 15; 17
required including failure prevention, (chapters 7, 9, 10)
mitigation, and recovery
Key Terminology
2. Change management: Change An understanding of the list of terms on pages 11-
management requires understanding 22 of this document is strongly recommended.
the strategic elements of successful The list is intended to be thorough but not
organizational change and how to apply exhaustive. The candidate is also expected to be
them. Principles and techniques familiar with the definitions of terms identified in
include: the content outline. Definitions of these terms can
be found in the APICS Dictionary, 15th edition.
a. Roles and responsibilities in leading
change Bibliography
b. Change process activities utilizing All test candidates should familiarize
typical project techniques themselves with the references for this
c. Preparing others for change and examination. The recommended references
understanding the needs and pertaining to the diagnostic areas are listed at
strategies to involve all stakeholders the end of each section of the content outline.
The first number indicates the sequence
3. Infrastructure systems: It is necessary number for the reference in the Bibliography
to understand how to configure various section on pages 9–10 and the numbers in
systems during implementation of the parenthesis indicate the relevant chapters
operations strategy. Principles and within that reference.
techniques include:

46 ©APICS
CPIM Part 2 Sample Questions 4. To create an effective customer-supplier
The following twelve questions are similar in partnership, it is essential to:
format and content to the questions on the
exam. These questions are intended for (A) establish stability in schedules that
practice and to illustrate the way questions enable suppliers to react within their
are structured. The degree of success you lead time.
have in answering these questions is not (B) evaluate the suppliers based on
related to your potential for success on the price and then send the purchase
actual exam and should not be interpreted as order.
such. (C) establish a delivery schedule with a
corresponding purchase order and
Read each question, select an answer, and send them to the suppliers.
check your response with the explanation on (D) establish a preventive maintenance
pages 52-54. program to avoid quality problems
due to machine troubles.
1. The forecast interval would typically be
longest for forecasts used as input to Use the information below to answer
which of the following processes? questions 5.

(A) Business planning Lead time: 2 Lot size: 30


(B) Final assembly scheduling (FAS) Demand time fence: 3 On hand: 15
(C) Sales and operations planning Planning time fence: 7 Safety stock: 6
(S&OP)
(D) Master production scheduling (MPS) Period 1 2 3 4 5
Forecast 10 22 20 24 28
2. In a distribution environment, which of the Customer 5 26 15 6 30
following will occur if the planner fails to orders
address exception messages during the Projected
planning cycles? available
balance
(A) Planned orders are not converted on time Available-to-
(B) Replenishment lead times are incorrect promise
(C) The statistical order point is increased Master
(D) The planning horizon is too short production
schedule
3. Cause-and-effect analyses are made
with the help of which of the following? 5. For the master schedule, what is the available-
to-promise for Period 4 if the discrete method
(A) Fishbone chart is used?
(B) Statistical process control (SPC)
method (A) 22
(C) Critical path method (CPM) (B) 24
(D) Pareto analysis (C) 35
(D) 37

CPIM Part 2 Content 47


6. Given the following purchase cost data 10. Which of the following is an advantage
for product Z: of using a lag capacity (chase) strategy
approach in implementing operations
0 on hand (December 27) strategy?
100 @ $10 = $1,000 (December 28)
10 @ $11 = $110 (January 3) (A) High facility utilization rate
10 @ $8 = $80 (January 10) (B) Revenue maximization
(C) Reaction to short-term demand
If this company is using a weighted changes
average costing method and 100 units (D) Product mix flexibility
were sold on January 8, the cost per unit
for the sale is: 11. Part number 762-4521 is being
processed as an order for a lot of 50
(A) $9.17 pieces due on day 200. The lot is
(B) $10.00 currently in queue at Operation 3. The
(C) $10.09 facility is currently running one eight-
(D) $8.00 hour shift. Based on routing and status
information for part number 762-4521
7. Which of the following is a significant below, what is the priority for Operation
factor in determining the level of work- 3 on Day 195 calculated on the basis of
in-process (WIP) inventory when a pull slack time rule (in days) per remaining
system is employed? operation?

(A) Number of open shop orders Operation Standard Pieces


(B) Quantity of parts represented by hours per produced
each signal piece through day
(C) Number of workstations in the 195
process 1 0.05 51
(D) Takt time required for the process 2 0.15 50
3 0.10 0
8. A make-to-order organization competing 4 0.20 0
on delivery speed would consider which 5 0.02 0
of the following to be a significant
impact on its competitive position? (A) -3.70
(B) 0.58
(A) Relocation of suppliers (C) 1.00
(B) Increased labor costs (D) 3.00
(C) Outsourcing of customer service
(D) Shifts in customer demand

9. Which of the following measures is a key


indicator of a firm’s asset utilization?

(A) Profit margin


(B) Current ratio
(C) Inventory turnover
(D) Cost of goods sold (COGS)

48 ©APICS
12.

The MRP Grid


Technique
Order quantity / Lot Size – lot-for-lot (L4L)
On hand: 500
Safety stock: 0
Allocated quantity: 0
Low-level code: 0
Lead time: 4
Periods
1 2 3 4 5 6 7 8
Gross requirements 100 300 200 400
Scheduled receipts
Projected available 500 400 100
x Net requirements
Planned order receipts
Planned order releases

The chart shows the gross requirements for


an item in a material requirements planning
system. Stock on hand is 500, and there is
nothing on order. The item has a lead time
of four periods and is being ordered lot-for-
lot. Which of the following would be the
correct planned order release for the item?

(A) 100 in Period 1, 400 in Period 3


(B) 100 in Period 4, 400 in Period 6
(C) 100 in Period 5, 400 in Period 7
(D) 500 in Period 5

CPIM Part 2 Content 49


history may not show the actual demand if
Answers to Sample product was unavailable for shipment.
Questions
6. B (IIIB) Resource requirements planning is
Note: References to the content outline long-range capacity planning completed at
appear in parentheses. the production plan level. A is incorrect
because capacity requirements planning is
CPIM Part 1 done at the MRP level. C is incorrect
because rough-cut capacity planning is
1. B (IG) A shipping buffer prevents missed done at the master schedule level. D is
due dates. A is incorrect because the incorrect because input/output control is
drum is the rate of production set by the completed during execution of the plan and
system’s constraint. C is incorrect is the shortest planning range.
because a constraint buffer is used to
buffer the constraint. D is incorrect 7. D (IIIB) To calculate how much actual
because the rope is the communications time will be needed to complete 16
process for releasing work. standard hours of work, divide the
capacity required by the efficiency times
2. B (IE1) The order of the five Ss approach the utilization (actual time = capacity
is sort, straighten, shine, standardize, required / (efficiency) (utilization) (actual
and sustain. A is incorrect because time = 16 / (80%) (80%). A is incorrect
sequence is not one of the five Ss. C is because it was incorrectly calculated by
incorrect because straighten is the multiplying the 16 hours required by 40
second step. D is incorrect because self- percent (the difference in the capacity
discipline is not one of the five Ss. and utilization from 100 percent) and
adding the amount to the 16 hours
3. A (IC1) A cash flow statement shows the required ((16 × 40% = 6.4), then (16 +
flow of cash and its timing into and out 6.4 = 22.4)). B is incorrect because it
of an organization. B is incorrect was incorrectly calculated by multiplying
because an income statement shows efficiency and utilization and then
profit and loss over a period of time. C is multiplying the required standard hours
incorrect because a balance sheet (80% × 80% = .64, then .64 × 16 =
shows the resources owned, the debts 10.24). C is incorrect because it is the
owed, and the owner’s equity at a given standard hours required and does not
point in time. D is incorrect because a consider the efficiency and utilization of
market share report indicates how well a the operation.
firm is doing in the market.
8. D (IVC3) Random-location storage
4. C (IIC2) Forecasts are usually wrong, enables parts to be placed in any space
therefore, every forecast should include that is empty. This method often
an estimate of error. A is incorrect required less storage space than a fixed-
because forecasts are more accurate for location storage method. A is incorrect
families or groups. B is incorrect because distribution is the activities
because forecasts are not used for associated with the movement of
independent demand items. D is material from the manufacturer to the
incorrect because forecasts are more customer. B is incorrect because kitting
accurate for near-term periods. is the process of constructing and
staging kits. C is incorrect because
5. D (IIC2) Intrinsic forecast data is based on access to stock is usually an advantage
interior factors such as sales history. A and of fixed-location storage.
B are incorrect because these are extrinsic
data. C is incorrect because shipment

50 ©APICS
9. C (IVC2) Private carriers lease or own 3. (IIIC3) The answer is data gathering,
their own transportation equipment. demand planning, supply planning,
Operating costs include not only pre-S&OP meeting, executive meeting.
investment in equipment, but insurance, The S&OP five-step process can be
permits and maintenance expenses as summarized as data gathering,
well. Most are company-owned and haul demand planning, supply planning,
only their own goods. A is incorrect pre-S&OP meeting and executive
because common carriers carry goods meeting. The step “Generate the MPS
for anyone wanting their services. B is is not part of the process.
incorrect because contract carriers haul
only for those with whom they have a 4. (IE1) The answer is sort, simplify,
contract. D is incorrect because parcel scrub, standardize, sustain.
carriers carry goods for the public. Five terms beginning with “S” used to
create a workplace suitable for lean
10. C (IVA1) Decoupling creates production: sort, simplify, scrub,
independence between supply and use standardize, sustain Sort means to
of material. A is incorrect because it is separate needed items from
lot size inventory. B is incorrect because unneeded ones and remove the latter.
it is inventory in transit between Simplify means to neatly arrange
locations. D is incorrect because hedge items for use. Scrub means clean up
is a form of inventory buildup to buffer the work area. Standardize means to
against some event that may not sort, simplify, and scrub daily. Sustain
happen. means to always follow the first four
Ss. Sometimes referred to by the
Answers to TEI Example Japanese equivalents: seiri, seiton,
seiso, seiketsu, and shitsuke
Questions
Technology Enhanced Items (TEI’s)

1. (IVA2) The inventory turnover is 4


times. A frequently used method to
compute inventory turnover is to
divide the annual cost of goods sold by
the average inventory level.

2. (IVB4) The answer is invoice, purchase


order, receiving report. The purchase
order, receiving report and the invoice
should be consistent regarding items
and quantities, so that the items and
quantities the supplier will be paid for
are consistent with the items and
quantities that were ordered and
received. The bill of lading, packing
list, and customs documentation can
be seen as information related to the
transport of the goods and not
impacting the invoice approval. The
purchase requisition precedes the
purchase order and is not required for
invoice approval.

CPIM Exam Content Manual 51


5. B (IIID) Available-to-promise for Period 4
Answers to Sample is the master production schedule (MPS)
Questions of 30 for that period, less the customer
orders of 6 for that period. Because
Note: References to the content outline another MPS of 30 will occur in Period
appear in parentheses. 5, demand for that period is not
considered by the available-to-promise
CPIM Part 2 for Period 4. Based on the explanation
for the calculation for discrete available
1. A (IB4) Business planning is performed to promise, Answers A, C, and D are
at the highest level of aggregation and incorrect.
over the longest horizon and would
typically be done for fiscal quarters or 6. C (IB) Standard costing is a method of
years. The other processes would have setting a new standard based on various
shorter horizons and intervals. parameters. A is incorrect because of
average cost after January 10. B is
2. A (IVB) Answers B, C, and D refer to incorrect because it is the FIFO cost of
system parameters that exception the product. C is correct because this is
messages would not address. Answer the weighted average on January 8. D is
(A) is a condition and exception incorrect because this is the last cost
messages address conditions paid for each item.

3. A (IIIB) The fishbone chart is a diagram 7. B (IIB) The level of work-in-process (WIP)
of the possible causes of a problem. The inventory when a pull signal is employed
causes are determined with the aid of is a function of the number of pull
brainstorming techniques. The diagram signals and the quantity represented by
resembles a fish skeleton. B, C, and D each pull signal. A, C, and D are
are incorrect. Statistical process control incorrect. Shop orders (A) are
(B) focuses on the continuous characteristic of a push system and are
monitoring of a process. The critical path not relevant in a pull system. The
method (C) is a technique used to plan number of workstations (C) in the
and control the activities of a project. process determines the minimum work
Pareto analysis (D) is a technique to in process (WIP) level, but does not
rank order the relative frequency of determine the total WIP level. The takt
occurrences. time (D) determines the rate at which
the process needs to operate. It does
4. A (IVA) Partnerships require a not determine the WIP level.
commitment to stability within the
agreed-upon response parameters. B 8. D (IIB2) As customer demand shifts, the
and C are short-term and tactical in organization may find itself with capacity
nature. D is something that needs to be in excess or less than required. Neither
done without any regard to partnering. situation is ideal. If the demand shift
leaves capacity short, we see a situation
where the operation does not have
sufficient capacity to meet market
needs.

52 ©APICS
9. C (IIE1) Profit margin is a profitability
measure (net income/sales). The
current ratio is a liquidity measure
(current assets/current liabilities). The
price-to earnings (P-E) ratio is a market
value measure (price per
share/earnings per share). Inventory
turnover (cost of sales/inventory level) is
one of the principal measures of a
company’s asset utilization.

10. A (IID1) A pure chase demand plan is


usually adopted by operations that
cannot store their output such as a call
center. Thus, the strategy avoids the
wasteful provision of excess staff that
occurs with a level capacity plan.

11. C (IB) The slack time per remaining


operation formula is:

Due date – date now – standard processing


days remaining

The result in divided by the number of remaining


operations.

Operation Standard Pieces Process


hours produced remaining
hours
1 0.05 51 0
2 0.15 50 0
3 0.10 0 5
4 0.20 0 10
5 0.02 0 1

The priority of Operation 3 is calculated as


follows:

CPIM Exam Content Manual 53


12. A (IIC)

The MRP Grid


Technique
Order quantity / Lot Size – lot-for-lot (L4L)
On hand: 500
Safety stock: 0
Allocated quantity: 0
Low-level code: 0
Lead time: 4
Periods
1 2 3 4 5 6 7 8
x Gross requirements 100 300 200 400
Scheduled receipts
Projected available 500 400 100
Net requirements 100 400
Planned order receipts 100 400
Planned order releases 100 400

54 ©APICS
apics.org Stock #09051-V60

Вам также может понравиться